Pulm Flashcards

1
Q

What are complications of systemic miliary TB

A
  1. Kidneys
  2. Meninges in Brain
  3. Lumbar vertebrae - POTT disease
  4. Adrenal glands: Addison’s disease
  5. Liver: hepatitis
  6. Cervical lymph nodes:
How well did you know this?
1
Not at all
2
3
4
5
Perfectly
2
Q

Treatment for active TB

A

RIPE:

Treatment - combination of rifampin, isoniazid,
Pyrazinamide, ethambutol 6-9 months

Prophylaxis - Rifampin or isoniazid - 9 months

How well did you know this?
1
Not at all
2
3
4
5
Perfectly
3
Q
TLC
IRV
TV
IC
ERV
RV
FRC
VC
A
  1. TLC- Total Lung Capacity- Volume of gas present in lungs after a maximal inspiration
  2. IRV-Inspiratory reserve volume (not labeled)- Air that can still be breathed in after normal inspiration
  3. TV- Tidal Volume- Air that moves into lung with each quiet inspiration, typically 500 mL
  4. IC-Inspiratory Capacity- IRV+TV
  5. ERV- Expiratory Reserve Volume- Air that can still be expelled after normal expiration
  6. RV- -Residual Volume- Air in lung after maximal expiration; cannot be measured on spirometry
  7. FRC-Functional Residual Capacity- RV+ERV (volume in lungs after normal expiration)
  8. VC-Vital Capacity-TV+IRV+ERV
How well did you know this?
1
Not at all
2
3
4
5
Perfectly
4
Q

FEV1

A

FEV1, which is the amount of air forcibly exhaled in the first second after maximal inspiration, is a good measure of airflow. When expressed as a % of the forced VC (FVC), it can be used to diagnose obstructive diseases such as COPD and asthma. Normal FEV1% > 80.
Restrictive lung disease patients demonstrate a normal or increased FEV1%. FVC and FEV1 are both decreased.

How well did you know this?
1
Not at all
2
3
4
5
Perfectly
5
Q

How do each of these parameters change for
obstructive (emphysema)
restrictive (fibrosis)

TLC 
FEV1
FVC
FEV1/FVC
Peak flow 
FRC
RV
A
Obstructive: 
TLC - increases 
*FEV1 - really decreases
FVC - decreases
FEV1/FVC - decreases
Peak flow - decreases
FRC - increases 
RV - increases 
Restrictive:
*TLC - really decreases
FEV1 - decreases
*FVC - really decreases
FEV1/FVC - increases/normal
Peak flow - decreases
FRC - decreases
RV - decreases
How well did you know this?
1
Not at all
2
3
4
5
Perfectly
6
Q

Where is the most of airway resistance and autonomic control

A

Resistance = 1/radius4

In the branching airway system of the lungs, the first and second order bronchi represent most of the airway resistance:

Sympathetic adrenergic neurons activate β2 (Gs) receptors on bronchial smooth muscle, leading to relaxation and dilation of the airways via increases in cAMP. Parasympathetic cholinergic neurons (vagal branches) activate muscarinic receptors (M3- Gq), which lead to contraction and constriction of the airways via increases in intracellular [Ca2+].

  • Asthma and COPD (chronic bronchitis and emphysema)- Many inflammatory mediators also cause bronchoconstriction by acting on specific receptors, all of which are linked to increases in intracellular Ca2+ and contraction via the PLC and DAG pathway. Short and long-acting β2 agonists are used to counteract bronchoconstriction. Vagal tone to the lungs can also be reduced with anti-cholinergic drugs.
How well did you know this?
1
Not at all
2
3
4
5
Perfectly
7
Q

What is minute ventilation (VE) and formula

What is alveolar ventilation (VA) and formula

Normal values for RR, Vt, Vd

A

Ve=total volume of gas entering the lungs per minute
Vt x RR

Alveolar ventilation - VA - volume of gas per unit time that reaches the alveoli
(Vt - Vd) x RR

INcreases in Va results in decrease in PCO2 while a decrease in Va results in PCO2

Normal values: Respiratory Rate=12-20 breaths/min
VT = 500 mL/breath
Vd = 150 mL/breath

How well did you know this?
1
Not at all
2
3
4
5
Perfectly
8
Q

There are two ways to increase respiration. One way is to increase the respiratory rate (RR). The other way is to increase tidal volume (VT). Assume a baseline RR of 15, a VT of 500 ml, and a dead space volume of 150 mls. Then double minute ventilation by one of the above methods. To increase alveolar respiration, which method is superior or are they equivalent?
A. Increased RR better
B. Increased VT better
C. Methods equivalent

A

Answer B - better to breath deeper than faster

Minute ventilation goes from 7,500 ml/min to 15,000 ml/min in either case

Va with increase in RR
Va = (Vt-Vd) x RR = 350 x 30 = 10,500 ml/min

Alveolar ventilation with increase Vtidal
Va = (Vt - Vd) x RR = 1000 x 15 = 15,000 ml/min

How well did you know this?
1
Not at all
2
3
4
5
Perfectly
9
Q

What is physiological deadspace?

A

Ventilated but not perfused.

Vd = anatomic deadspace (conducting airways + functional dead space in alveoli

Apex of healthy lung is largest contributor of functional dead space. An ideal lung has no functional dead space.

PaCO2 = arterial PCO2
PECO2 = expired air PCO2 

Vd = Vt x [(PaCO2 - PECO2)/PaCO2]

If no alveoli participate in gas exchange (not perfused) then there is no expired CO2 (PeCO2) and Vd = Vt and everything is deadspace. If expired Co2 equals arterial CO2 then Vd = 0 and there is no dead space. Neither extreme exists in life .

How well did you know this?
1
Not at all
2
3
4
5
Perfectly
10
Q

In the measurement of physiologic dead space using Bohr’s method, the arterial and expired pCO2 were 40 and 30 respectively. What is the ratio of deadspace to tidal volume?

A

Vd = Vt x [(PaCO2-PeCO2)/PaCO2]

0.25 is answer

How well did you know this?
1
Not at all
2
3
4
5
Perfectly
11
Q

A man who has a tidal volume of 500 mL is breathing at a rate of 15 breaths/min. The PCO2 in his arterial blood in 40 mm Hg and the PCO2 in his expired air is 30 mm Hg. Whwat is his minute ventilation?
What percentage of each tidal volume is dead space? What percentage of each tidal volume reaches functioning alveoli? What is his alveolar ventilation?

A

What is his minute ventilation?
500 x 15 = 7500 ml/min

What percentage of each tidal volume is dead space?
Vd= Vt x (PaCO2- PECO2)/PaCO2= 500 x 10/40 = 125 ml or 25%

What percentage of each tidal volume reaches functioning alveoli? 75%

What is his alveolar ventilation? (500-125) x 15 = 5625 ml/min

How well did you know this?
1
Not at all
2
3
4
5
Perfectly
12
Q

Describe structure of hemoglobin- what is special about fetal

A

4 polypeptide units (2 α and 2 β) and 2 forms;

1) T (taut) form has low affinity for O
2) R (relaxed) has high affinity for O

Taut in Tissues, Relaxed in Respiratory

Fetal hemoglobin (2α and 2F subunits) has lower affinity for 2,3-BPG than adult Hb and thus has a higher affinity for O2, driving diffusion of oxygen across placenta to fetus. Hemoglobin acts as a buffer for Hydrogen ions.

How well did you know this?
1
Not at all
2
3
4
5
Perfectly
13
Q

Why is the dissociation curve of oxygen-hemoglobin sigmoidal shaped.

WHat factors shift towards left
What factors shift toward right?

A

Sigmoidal shape due to positive cooperativity (higher affinity for each subsequent oxygen molecule bound). Myoglobin is monomeric > no positive cooperativity and not sigmoidal. (hyperbolic)

When curve shifts to the right, decreased affinity of hemoglobin for O2 (t form). An increase in all factors listed causes a shift of the curve to the right.

Left:

  • decreased temperature
  • decreased 2-3, BPG
  • increase in pH (decreased (H+),
  • CO

Right shift

  • increased temp
  • increased CO2
  • increased 2,3 BPG
  • decrease in pH - acidosis
How well did you know this?
1
Not at all
2
3
4
5
Perfectly
14
Q

Equation of Oxygen content and oxygen delivery.

also normal values for
O2 content, Hb content, binding ability.

A

O2 content = 1.34 x SaO2 x Hb + (PaO2 x .0003)

The .0003 shows that very little oxygen can be dissolved in blood.
normally 1 g Hb can bind 1.34 mL O2
Normal Hb amount in blood is 15 g/dL

O2 binding capacity is normally around 20

O2 delivery to tissues = cardiac output x O2 content of blood.

How well did you know this?
1
Not at all
2
3
4
5
Perfectly
15
Q

How do each of these affect

Hb level
% O2 saturation
Dissolved O2
Total O2 content

  1. CO poisoning
  2. Anemia
  3. Polycythemia
A
1. CO poisoning
Hb level - normal 
% O2 saturation - decreases
Dissolved O2 - normal 
Total O2 content -decreases
2. Anemia 
Hb level - decreases 
% O2 saturation - normal
Dissolved O2 - normal 
Total O2 content -decreases
*reduced O2 content, normal saturation 
3. Polycythemia 
Hb level - increase
% O2 saturation - normal
Dissolved O2 - normal 
Total O2 content -increase
How well did you know this?
1
Not at all
2
3
4
5
Perfectly
16
Q

Describe CO2 transport - pulm

A

Carbon dioxide is transported from tissues to the lungs in 3 forms:

1) Bicarbonate (90%)- vast majority is generated in RBCs and put into plasma in exchange for chloride. H+ buffered in RBCs by Hb
2) Carbaminohemoglobin or HbCo2 (5%) Co2 bound to Hb at N-trminus of the globin (not heme). Co2 binding favors taut form (O2 unloaded)
3) Dissolved CO2 (5%) contributes to serum pH.

How well did you know this?
1
Not at all
2
3
4
5
Perfectly
17
Q

Ficks Law’s of diffusion

A

Depends on

  1. surface area
  2. thickness
  3. Solubility of gas
  4. Pressure difference

Even though Co2 has a greater MW than O2, its diffusion is about 20x greater because CO2 is highly soluble in blood!

How well did you know this?
1
Not at all
2
3
4
5
Perfectly
18
Q

What are normal values for the following

Tracheal PO2

Alveolar PO2

Alveolar PCO2

at end of gas exchange

A

Tracheal PO2 - 150 mm Hg (atmospheric)

Alveolar PO2 - 100 mmHg

Alveolar PCO2 - 40 mmHg

Poor alveolar perfusion would increase PO2 and decrease PCO2.

How well did you know this?
1
Not at all
2
3
4
5
Perfectly
19
Q

Equation for PaCO2

A

PaCO2 = [VCo2 x .863]/V

Alveolar ventilation is inversely related to alveolar PCO2.

How well did you know this?
1
Not at all
2
3
4
5
Perfectly
20
Q

What are the effects on
pH
Bicarb
PaCO2 and compensation in

Metabolic acidosis
Metabolic alkalosis
Respiratory acidosis
Respiratory alkalosis

A
  1. Metabolic acidosis - pH goes down, bicarb goes down
    Compensation - PaCO2 decreases - hyperventilation
  2. Metabolic alkalosis - pH goes up, bicarb goes up,
    Compensation: PaCo2 increases -hypoventilation
  3. Respiratory acidosis - pH decreased, increased PaCO2,
    Compensation: increased bicarb
  4. Respiratory alkalosis - pH increased, decreased PaCo2,
    Compensation: drop in bicarb
  5. Respiratory alkalosis
How well did you know this?
1
Not at all
2
3
4
5
Perfectly
21
Q

What are the values for
in acute respiratory acidosis

pH-7.4, PCO2 = 40, HCO3 = 24

For every 10 PaCO2 increase

  • pH decrease
  • HCO3 increase

Chronic respiratory acidosis

For every 10 PaCo2 increase

  • pH decrease
  • bicarb increase
A

For every 10 PaCO2 increase

  • **-pH decrease - 0.08
  • HCO3 increase - 1

Note if bicarb is above 30 look for another process like chronic respiratory acidosis

Chronic respiratory acidosis

For every 10 PaCo2 increase

  • **-pH decrease - 0.03
  • bicarb increase - 3-4 (renal compensation)

-Renal compensation maximum in 4 days, PH never normalized

How well did you know this?
1
Not at all
2
3
4
5
Perfectly
22
Q

What are the values for
in acute respiratory acidosis

pH-7.4, PCO2 = 40, HCO3 = 24

For every 10 PaCO2 decrease

  • pH increase
  • HCO3 decrease

Chronic respiratory acidosis

For every 10 PaCo2 decrease

  • pH increase
  • bicarb decrease
A

For every 10 PaCO2 decrease

  • **-pH increase- 0.08
  • HCO3 decrease - 2

Relationship is true down to a PCO2 of 20

Chronic respiratory acidosis

For every 10 PaCo2 decrease

  • **-pH increase - 0.03
  • bicarb decrease - 5 renal compensation)

Holds down to a PCO2 of 20, can be complete - renal compensation unlike acidosis

How well did you know this?
1
Not at all
2
3
4
5
Perfectly
23
Q

What is winters formula?

A

Applies to metabolic acidosis

Expected PCO2 = (1.5 x serum HCO3-) + (8+/- 2)

If PCO2 is lower than expected there is a concomitant respiratory alkalosis.

Compensation for a metabolic alkalosis

PCO2 would increase by 5 mm Hg for every 10 increase in HCO3 but PCO2 rarely increases above 50 mm Hg

How well did you know this?
1
Not at all
2
3
4
5
Perfectly
24
Q

Diffusion capacity - A/T x D x change in pressure

What factors increase and decrease these values

A

Decreased by:

  1. lung fibrosis (increase T),
  2. emphysema (decreased A),
  3. pulmonary hypertension/embolism (decreased blood flow from increased resistance),
  4. CHF (decreased blood flow) and
  5. anemia (decreased RBCs)

Increased (all increase change in P) by 1. polycythemia (more RBCs),

  1. exercise (increased pulmonary blood flow),
  2. intracardiac L to R shunts (increased pulmonary blood flow)
How well did you know this?
1
Not at all
2
3
4
5
Perfectly
25
Q

How do PO2 levels affect pulmonary and systemic circulations?

A

PO2 exerts opposite effects on pulmonary and systemic circulation. In the lungs, a decrease in PO2 causes hypoxic vasoconstriction, shifting blood away from poorly ventilated to well-ventilated regions. In the systemic circulation, hypoxia promotes vasodilation not vasoconstriction

hypoxia promotes vasodilation to get oxygen to tissues. Constriction is to divert blood away.

It is important for the blood to be distributed to those segments of the lungs where the alveoli are best oxygenated. When the oxygen tension of the alveoli decreases below normal, the adjacent blood vessels constrict causing their resistance to increase as much as fivefold at extremely low oxygen levels. This is opposite to the effect observed in systemic vessels, which dilate in response to low oxygen (opposite of pulmonary vessels).

How well did you know this?
1
Not at all
2
3
4
5
Perfectly
26
Q

Why do high and low lung volumes raise PVR

A

At high volumes, pressure on alveolar vessels causes them to collapse, raising PVR.

At low volumes, traction on extra- alveolar vessels is absent and vessels collapse, raising PVR.

The lowest PVR is at FRC.

How well did you know this?
1
Not at all
2
3
4
5
Perfectly
27
Q

How to calculate PAO2

A

PAO2 = .21 (Patm - 47 mmHg) - (PaCO2/.8)

How well did you know this?
1
Not at all
2
3
4
5
Perfectly
28
Q

V/Q mismatch in the lung - normal

Pa, Pv, Pa for all three zones

A

Ideally, ventilation is matched to perfusion (V/Q=1) for adequate gas exchange

Lung zones:
Apex of lung- V/Q=3 (wasted ventilation)
Base of lung- V/Q=0.6 (wasted perfusion)

V/Q > 0 = airway obstruction (shunt). In shunt, 100% O2 does not improve PO2 (eg A-v malformations)

V/Q > infinity = blood flow obstruction (physiological dead space). 100% O2 does improve PO2 assuming <100% deadspace

Zone 1 - PA> Pa>Pv (no blood flow -alveolar is better)
Zone 2 - Pa>PA > Pv
Zone 3 - Pa > Pv > Pa

How well did you know this?
1
Not at all
2
3
4
5
Perfectly
29
Q

What are the signs and symptoms of high altitude pulmonary edema aka HACE

When do symptoms occur?

A

Symptoms: shortness of breath, dry cough, fever, reduced exercise performance, weakness and lethargy

36-72 hours after arrival.

Physical signs 
-tachypnea
-tachycardia
-cyanosis 
\+/- rales 
-loud P2, RV heave
How well did you know this?
1
Not at all
2
3
4
5
Perfectly
30
Q

What are the three compensatory mechanisms for high altitude

A
  1. Immediate -Hyperventilation leading to respiratory alkalosis - decreased atmospheric pressure decreases alveolar oxygen tension. To compensate, decrease alveolar carbon dioxide levels would correspond to an increase in PAO2. The alkalosis, increased pH, favors oxygen loading at the alveoli
  2. Days: Increased 2,3 DPG - doesn’t shift the whole curve but shifts the setpoint to the right. Increases oxygen unloading in tissues
  3. Days to weeks
    Kidneys increase level of erythropoietin, which stimulates he bone marrow to increase RBC and hemoglobin levels> thereby increasing oxygen content of blood.
How well did you know this?
1
Not at all
2
3
4
5
Perfectly
31
Q

What are three classes of drug for high altitude prophylaxis

A
  1. Nifedipine
  2. Sildenafil
    * *3. Acetazolamide - two days before a hike. It gives you a metabolic acidosis so when you are on the hike you can exceed levels of ventilation you could’ve had without it.
    - inhibiting bicarb reabsorption
How well did you know this?
1
Not at all
2
3
4
5
Perfectly
32
Q

Tracheobronchial source is most common for hemoptysis - what are the 3 B’s?

A
  1. Bronchitis (acute or chronic) - 60-70% of cases
  2. *(Bronchogenic carcinoma), endobronchial metastatic tumor, Karposi’s sarcoma, bronchial carcinoid
    - 23% of cases
  3. Bronchiectasis (inflammation of airways; cystic fibrosis)
How well did you know this?
1
Not at all
2
3
4
5
Perfectly
33
Q

What is Kussmaul’s breathing?

A

Increased rate and DEPTH of breathing

This is associated with *ACIDOSIS

Hyperpnea

How well did you know this?
1
Not at all
2
3
4
5
Perfectly
34
Q

What is Cheyne Stokes breathing?

A

FA: May be associated with Central Sleep apnea

Constant rate of breathing with variable depth include apneic period (aka suspension of breathing)

Causes:

  1. neurologic disorder (cerebral hemorrhage)
  2. **Congestive heart failure (around 40% of CHF patients will have this kind of breathing at night
  3. High altitude and normal aging
How well did you know this?
1
Not at all
2
3
4
5
Perfectly
35
Q

What are the accessory muscles of respiration

A

Expiration

  1. Abdominal muscles (rectus abdominis)
  2. INternal intercostals

Inspiration:

  1. Sternocleidomastoid
  2. Scalenes
  3. Trapezius
  4. EXternal intercostals

In COPD, it is difficult for air to get out. Therefore you need to use these accessory muscles.

If a patient is using their accessory muscles and has obstructive pattern on pulmonary test they likely have COPD.

How well did you know this?
1
Not at all
2
3
4
5
Perfectly
36
Q

What is paradoxical breathing?

A

Associated with respiratory fatigue or diaphragmatic weakness

So basically with normal breathing the abdomen extends outwards with chest expansion.

In this case the abdomen goes inward. The accessory muscles are pulling allowing the chest to rise but pulled with it is the diaphragm, pulling the abdominal contents up with it.

How well did you know this?
1
Not at all
2
3
4
5
Perfectly
37
Q

Emphysema “pink puffer”

A
  1. α1-Antitrypsin deficiency results in excess elastase activity, which can cause emphysema. (can be congenital or from free radicals of tobacco smoke or neutrophils)
    - In lungs, reduction in α1-antitrypsin leads to uninhibited elastase in alveoli leading to reduced elastic tissue > panacinar emphysema.
  2. Centriacinar—associated with
    smoking. Frequently in
    upper lobes (smoke rises up). Panacinar—associated with
    α1-antitrypsin deiciency.
    Frequently in lower lobes.
  3. Enlargement of air spaces, decreased recoil, increased compliance, decreased DLCO from destruction of alveolar walls.
    INcreased elastase activity > increasedloss of elastic fibers >increasedlung compliance.
CXR: increase AP diameter, flattened
diaphragm, increasdlung ield
lucency.
Barrel-shaped chest 
Exhalation through pursed lips
to increase airway pressure
and prevent airway collapse.

[class notes]
Trapped air—hyperinflation and overdistention
•  As more alveoli coalesce, blebs and bullae may develop
•  Destruction of alveolar walls and capillaries—reduced surface
area for O2 diffusion (hence the major difference with asthma which is that emphysema has a diffusion problem too)
•  Compensation is done by increasing respiratory rate to
increase alveolar ventilation
•  Hypoxemia and hypercapnea usually develops late in disease - results from hypoventilation and increase airway resistance and problems with alveolar gas exchange)
•  Minimal coughing with no to small amounts of sputum
•  Overdistention of alveoli causes diaphragm to flatten and AP
diameter to increase

Complications
•  Pulmonary hypertension (pulmonary vessel constriction d/t
alveolar hypoxia and acidosis)
•  Cor pulmonale (Rt heart hypertrophy +- RV failure)
•  Pneumonia
•  Acute Respiratory Failure

•  V/Q mismatch
•  Emphysema: increase ventilation of poorly perfused lung units (high V/Q ratio) increased physiological dead space.
*-the area is getting destroyed so blood flow will not be sent there with vasoconstriction. You are ventilating areas without blood flow, this is why pulmonary hypertension comes later in the disease
•  Chronic bronchitis: perfusion of under-ventilated areas, low V/
Q ration and subsequent physiological shunt.
*mucus pus in alveolar space, what is happening to your ventilation and perfusion. You are having areas poorly ventilated but still perfusing. That is what we call a shunt like hypoxia.

How well did you know this?
1
Not at all
2
3
4
5
Perfectly
38
Q

If you hear bronchial breath sounds, what should you think of?

A

Pneumonia

How well did you know this?
1
Not at all
2
3
4
5
Perfectly
39
Q

What conditions do you think of when you hear

Fine crackles

A

***interstitial lung disease

also pneumonia, CHF

How well did you know this?
1
Not at all
2
3
4
5
Perfectly
40
Q

How do fine crackles and coarse crackles differ

A

Fine crackles

  • altered by body position change but remains unaltered by coughing
  • it is not transmitted to mouth
  • produced by sudden inspiratory opening of small airways which were held closed during a previous expiration
  • *interstitial lung disease

Coarse crackles

  • it is altered by coughing but not by body position changes
  • it can be transmitted to mouth
  • produced by gas passing through airways which undergo intermittent opening and closing.
How well did you know this?
1
Not at all
2
3
4
5
Perfectly
41
Q

Describe what would be seen with each of the physical findings for:

  1. Breath sounds
  2. Percussion
  3. Fremitus
  4. Tracheal deviation
Pleural effusion 
Atelectasis 
Simple pneumothorax 
Tension pneumothorax
Consolidation (lobar pneumonia, pulmonary edema)
A
  • Pleural effusion
    1. Breath sounds -decreased
    2. Percussion -dull
    3. Fremitus - decrease
    4. Tracheal deviation: either none or away from side of lesion
  • Atelectasis
    1. Breath sounds - decreased
    2. Percussion -dull
    3. Fremitus - decrease
    4. Tracheal deviation -toward side of lesion
  • Simple pneumothorax
    1. Breath sounds - decreased
    2. Percussion -hyper-resonant
    3. Fremitus - decrease
    4. Tracheal deviation - none
  • Tension pneumothorax
    1. Breath sounds - decreased
    2. Percussion - hyper-resonant
    3. Fremitus - decrease
    4. Tracheal deviation - away from side of lesion
  • Consolidation (lobar pneumonia, pulmonary edema)
    1. **Breath sounds - bronchial breath sounds, late inspiratory crackles,
  • egophony: Eee > Aaa change, associated with pleural effusion,
  • bronchophony: 99 if heard with increased clarity and distinction,
  • whispered pectoriloquy: whispered sounds heard clearly and distinctly
    2. Percussion - dull
    3. Fremitus - increase
    4. Tracheal deviation - none
How well did you know this?
1
Not at all
2
3
4
5
Perfectly
42
Q

The critical feature of her case is that Mrs. Smith, age 35, has been having “asthma attacks” since her early childhood. Her attacks are characterized by the relatively sudden onset of dyspnea; they are more frequent in the spring and fall, when they are often preceded by symptoms of rhino-conjunctivitis. If untreated, an attack will last for a few days, but if she is treated with a subcutaneous injection of adrenaline, as you have administered at your office, she often has relief from acute symptoms, and the attack may or may not recur. Recently, her attacks have been more frequent, and she does not feel that her breathing is improved to the point where she can carry out her responsibilities as a wife and mother.

A

Asthma

How well did you know this?
1
Not at all
2
3
4
5
Perfectly
43
Q

Following transfusion, oxygen delivery will improve leading to a rise in arterial and mixed venous oxygen content (so OK, I think we are all good with that idea!) Transfusion will not change the arterial PO2 (OK, makes sense) and not change tissue oxygen consumption

A

I said low O2 delivery, low mixed venous oxygen (MVO2) content- this is true and I may have said the tissues take more O2 etc. but to clarify- LOW O2 DELIVERY results in an INCREASE IN O2 EXTRACTION not in the use or consumption of oxygen, which reflects the metabolic rate of the tissue. The removal of an increased fraction of the O2 from the blood will result in LOW MVO2.

To summarize, there are two main concepts linked to the idea of O2 content in mixed venous blood (MVO2).

The first concept is related to oxygen delivery. ANY decrease in O2 delivery to the tissue will increase the extraction of oxygen from the blood (and by doing so, will decrease MVO2). The tissue will extract more oxygen from the blood in order to meet its metabolic needs (i.e. its rate of oxygen utilization)- so in fact, the RATE of O2 USE does not change, but % of O2 removed from the blood per 100 ml WILL INCREASE because less O2 is coming in. Thus, if you RESTORE O2 delivery, O2 extraction rate will return to baseline- but the use of O2 by the tissue- its metabolic rate, remains the same. We can make up some math for this: Let’s say I NEED 32 oz. of coffee a day- and 64 oz. is regularly delivered- so I routinely take 50% of all the coffee coming to me. Well, one day, someone takes some of my coffee (grrrr! probably Dr. Foty)- and only 48 oz. gets to me- well, I still need my 32 oz- but if I take my regular 50%- I won’t get what I need- so I take 75% this time. My needs haven’t changed- but I need to increase my “extraction” of coffee-then, if the next day, the whole 64 oz. comes- I will go back to my 50%. So my coffee consumption does not change- only the fraction that I remove from the pot. I hope that makes sense. To problem solve for things that change MVO2 in this way, you want to think about what would be the causes of low oxygen delivery - think about the equation: DO2 (delivery) = Cardiac Output X O2 content of the blood (CaO2). As we have discussed, CaO2 is dependent on the the Hgb, O2 SAT and PaO2. So decreased CO, Hgb, O2 SAT and PaO2 will all result in a decrease in MVO2.

The other main concept linked to MVO2 is oxygen consumption by the peripheral tissue (this is O2 USAGE- not to be confused with EXTRACTION)- so anything that increases (e.g. fever, exercise) or decreases (e.g.cyanide, hypothermia) tissue oxygen consumption (that is the USE of O2) will ALSO change the mixed venous O2- in this case, an increase in tissue O2 USE (i.e. metabolic rate) will decrease MVO2, while a decrease in tissue O2 use will increase MVO2. (so that is like, I am getting my 64 oz. delivered, but I decide to drink 40 oz today!)

How well did you know this?
1
Not at all
2
3
4
5
Perfectly
44
Q

The mechanism of pulmonary cystic fibrosis (sweat glands)

A

FA:
Autosomal RECESSIVE defect in CFTR gene on chromosome 7; commonly a deletion of Phe508.
Most common lethal genetic disease in Caucasian population.
__________________________
Patho:
CFTR encodes an ATP-gated Cl− channel that secretes Cl− in lungs and GI tract, and reabsorbs
Cl− in sweat glands. Most common mutation > misfolded protein > protein retained in RER and not transported to cell membrane, causing reduced Cl− (and H2O) secretion; increased intracellular Cl− results in compensatory increased Na+ reabsorption via epithelial Na+ channels > increased H2O reabsorption > abnormally thick mucus secreted into lungs and GI tract. increased Na+ reabsorption also causes more negative transepithelial potential difference.
___________________________
Diagnosis:
Increased Cl− concentration (> 60 mEq/L) in sweat is diagnostic. Can present with contraction alkalosis
and hypokalemia (ECF effects analogous to a patient taking a loop diuretic) because of ECF
H2O/Na+ losses and concomitant renal K+/H+ wasting. Increasedimmunoreactive trypsinogen (newborn screening).
__________________
Complications:
1. Recurrent pulmonary infections (eg, S aureus [early infancy], P aeruginosa [adolescence]), chronic bronchitis and bronchiectasis > reticulonodular pattern on CXR, opaciication of sinuses.
2. Pancreatic insuficiency, malabsorption with steatorrhea, fat-soluble vitamin deiciencies (A, D, E,
K) leading to foul smelling feces
3. biliary cirrhosis, liver disease. Meconium ileus in newborns. Infertility in men (absence of vas deferens, spermatogenesis may be unaffected) and subfertility in women (amenorrhea, abnormally thick cervical mucus).
Nasal polyps, clubbing of nails.
_____________
Treatment:
Multifactorial: chest physiotherapy, albuterol, aerosolized dornase alfa (DNAse), and hypertonic
saline facilitate mucus clearance. Azithromycin used as anti-inlammatory agent. Ibuprofen slows
disease progression. Pancreatic enzymes for insuficiency.

Sweat glands
With the lungs, CFTCR is secreting chloride. What follows the chloride? Sodium. What follows sodium? Water. That is why the mucus is so sticky and thick because no water.

Lung epithelial cells - CFTCR
(cystic fibrosis transmembrane conductance regulator)

The two ions are chloride and sodium but CFTCR is a chloride channel.

Has a big role in the sweat glands. when sweat is secreted there is a lot of sodium and chloride in the lumen. As it goes through the lumen, there are channels along the way to reabsorb. CFTCR are the chloride channels here, chloride will be reabsorbed into the body (as sodium does through their channels, balanced). So there are few sodium and chloride ions remaining in sweat, when we sweat alot the sweat becomes actually saltier because we its faster.

CF - no problem with sodium channel, chloride channels are not open so it can’t leave and keeps going towards the skin, Negative and positive want to stay together. Sodium is pulled by the chloride so alot more is in the sweat now. Salty skin is a symptom of CF.

How well did you know this?
1
Not at all
2
3
4
5
Perfectly
45
Q

Describe how flow volume loop changes in these situations

Obstructive
Restrictive
Fixed Obstruction
Variable extrathoracic
Variable intrathoracic
A

Obstructive - In obstructive disease, flow is reduced during expiration (scooped out curve) resulting in air trapping and an increase in RV.
-due to premature closing of those airways, loop shifts to the left. Inspiration is relatively normal

Restrictive - In restrictive disease, flow is normal during expiration resulting in a normal or increased slope in the expiratory phase. The problem here is the lung is stiff and all lung volumes are reduced. Loop shifts to right and is smaller everywhere

Fixed Obstruction - Fixed obstruction somewhere in large airways affecting flow in both the inspiratory and expiratory phases
-flat inspiration and expiration, looks like a rectangle sort of

Variable extrathoracic - Obstruction is extrathoracic and only occurs in inspiration when pressure in trachea becomes negative relative to atmospheric pressure

Variable intrathoracic - Obstruction is intrathoracic and only occurs during expiration when pressure in lung becomes greater than internal large airway pressure

How well did you know this?
1
Not at all
2
3
4
5
Perfectly
46
Q

What is flail chest?

A

Flail chest is a life-threatening condition that occurs when the rib cage is broken in several places due to trauma and becomes detached from the rest of the chest wall. Classic cause of paradoxical chest movement.

Inspiration - broken chest wall goes in due to negative pressure

Expiration

How well did you know this?
1
Not at all
2
3
4
5
Perfectly
47
Q

A 26-year-old female has the following arterial blood findings:
PO2 - 94 mmHg
%O2 saturation - 50
Oxygen content - 13 ml/dL

Which of the following is the most likely cause of the findings described above? A. Cyanide intoxication B. Morbid obesity C. High altitude D. Chronic blood loss E. Asthmatic attack F. carbon monoxide

A

F. carbon monoxide

Cyanide effectively and rapidly halts mitochondrial oxidative phosphorylation by binding to the ferric iron (Fe3+) of cytochrome oxidase a3. Affected cells can’t use oxygen for aerobic respiration, resulting in high anion-gap lactic acidosis - and as we have already discussed, one of the clues will be a RISE in the mixed venous oxygen content.

While cyanide does bind to the ferrous form (Fe2+) of iron found in oxyhemoglobin - it DOES NOT bind to the Fe2+ in oxyhemoglobin very well; it prefers Fe3+ (ferric form; methemoglobin)- which circulates in the blood in small amounts (range is 0-3% of total hemoglobin)- and therefore WHEN cyanohemoglobin IS measured in the blood, it really reflects the binding of cyanide to the ferric form of iron in methemoglobin primarily. For this reason, you would not expect cyanide to cause the O2 sat to drop in such a large way- Step 1 wants you to know that the problem with cyanide is in the periphery with the ability of the tissues to USE the oxygen that is delivered. I also attached a cyanide cartoon- it is a “sketchy”.

How well did you know this?
1
Not at all
2
3
4
5
Perfectly
48
Q

When it comes to metabolic acidosis check anion gap. What are the values to note and how to calculate?

MUDPILES
HARDASS

A

Anion gap = (Na+) - (Cl- + HCO3-)

> 12 mEq/L is elevated anion gap
8-12 is normal anion gap

MUDPILES - metabolic acidosis with elevated anion gap

  • methanol (formic acid)
  • uremia
  • diabetic ketoacidosis
  • propylene glycol
  • iron tablets or INH
  • lactic acidosis
  • ethylene glycol
  • salicylates
Normal anion gap metabolic acidosis
HARDASS
Hyperalimentation
(Addison disease - primary adrenal insufficiency/hypocortisolism)
Renal tubular acidosis
Diarrhea
Acetazolamide
Spironolactone
Saline infusion
How well did you know this?
1
Not at all
2
3
4
5
Perfectly
49
Q

RIPE for assessing image quality?

A

Rotation
• The medial aspect of each clavicle should be equidistant from the spinous
processes • The spinous processes should also be in vertically orientated against the
vertebral bodies.
should be visible
projected within the chest, it’s PA.

Inspiration
• 5-6 anterior ribs, the lung apices, both costophrenic angles and lateral rib edges

Projection
• AP vs PA film
• Tip- if there is no label, then assume it’s a PA. Also, if the scapulae are not •

Exposure
• Left hemidiaphragm visible to the spine and vertebrae visible behind heart

How well did you know this?
1
Not at all
2
3
4
5
Perfectly
50
Q

Chronic bronchitis (blue bloater)

A

PRESENTATION
Findings: wheezing, crackles, cyanosis (hypoxemia due to shunting), dyspnea, CO2 retention, 2° polycythemia.

PATHOLOGY Hypertrophy and hyperplasia of mucus-secreting glands
in bronchi >Reid index (thickness of mucosal gland layer to thickness of wall between epithelium and cartilage) > 50%.

OTHER Diagnostic criteria: productive
cough for > 3 months in a
year for > 2 consecutive years.

[Class notes]:
Elimination: Difficulty in clearing secretions because of poor ciliary
function, distal airway occlusion and ineffective cough
secondary to respiratory muscle weakness and reduced peak
expiratory flow

Production: Airflow obstruction: increased mucus hypersecretion causes luminal occlusion,epithelial layer thickening encroaches on the
airway lumen and increased mucus alters airway surface
tension, thereby predisposing it to expiratory collapse

Inflammation:
Inhaled cig smoke and noxious stimuli •  Increase neutrophils in airway lumen •  Macrophages in airway lumen, wall and parenchyma •  CD8 lymphocytes in airway wall and parenchyma (as opposed to CD4 in asthma)

How well did you know this?
1
Not at all
2
3
4
5
Perfectly
51
Q

Asthma (FA)

A
Findings: cough, wheezing, tachypnea, dyspnea, hypoxemia, decreasedinspiratory/
expiratory ratio, pulsus
paradoxus, mucus
plugging. 
Triggers: viral URIs, allergens,
stress. Diagnosis supported by
spirometry and methacholine
challenge

Bronchial hyperresponsiveness
> reversible bronchoconstriction. Smooth muscle hypertrophy and hyperplasia, Curschmann spirals (shed epithelium forms whorled mucous plugs), and *Charcot-Leyden crystals (eosinophilic, hexagonal, double-pointed, needle-like crystals formed from breakdown of eosinophils in sputum

Aspirin-induced asthma: COX
inhibition >leukotriene
overproduction >airway
constriction. Associated with
nasal polyps
How well did you know this?
1
Not at all
2
3
4
5
Perfectly
52
Q

Bronchiectasis (FA)

A

Bronchiectasis

Findings: purulent sputum, recurrent infections, hemoptysis, digital clubbing.

Chronic necrotizing infection
of bronchi >permanently dilated airways.

Associated with bronchial
obstruction, poor ciliary
motility (eg, smoking,
Kartagener syndrome),
cystic fibrosis, allergic
bronchopulmonary
aspergillosis.
[class notes]
•  occurs in the context of
chronic airway infection
and inflammation
•  mild to moderate airflow
obstruction
•  Small airways in
bronchiectasis are
obstructed from an
inflammatory infiltrate in
the wall

Etiology (think infection)
•  Congenital or hereditary conditions: cystic fibrosis, intralobar
sequestration of the lung, immunodeficiency states, primary
ciliary dyskinesia and kartagener syndrome
•  Bronchial obstruction due to tumor, foreign bodies, and mucus
impaction
•  Other conditions such as rheumatoid arthritis, systemic lupus
erythematous, inflammatory bowel disease, and post-
transplantation (chronic lung rejection and chronic graft-
versus-host disease after bone marrow transplantation)

How well did you know this?
1
Not at all
2
3
4
5
Perfectly
53
Q

What are the symptomatic classifications of asthma severity (intermittent, mild, moderate, severe)

A

Intermittent: symptoms <2 days/wk
-exacerbations requiring oral systemic corticosteroids 0-1 times/yr
Step 1 treatment- consider low dose inhaled corticosteroids, As needed short acting beta agonists

Mild: symptoms >2 days/wk but not daily,
-exacerbations requiring oral systemic corticosteroids 2 times/yr
Step 2 treatment - low dose inhaled corticosteroids, leukotriene receptor antagonist, low dose theophylline, As needed short acting beta agonists

Moderate: symptoms daily
-exacerbations requiring oral systemic corticosteroids 2 times/yr
Step 3 treatment: Low dose ICS/LABA (long acting beta2 agonists)

Severe: symptoms throughout the day
-exacerbations requiring oral systemic corticosteroids 2 times/yr
Step 4 or 5 treatment: normal dose of ICS/LABA, add on treatment of tiotropium, anti-IgE, anti IL5, bronchial thermoplasty

*same time frames for usage of short acting beta agonists

How well did you know this?
1
Not at all
2
3
4
5
Perfectly
54
Q

Define restrictive work and elastic work

A

Resistive work: energy needed to move air through the conducting airways to the alveoli.

Elastic work: energy needed to expand the lungs against their own recoil and the recoil of their surroundings (e.g. chest wall, abdomen, etc.)

How well did you know this?
1
Not at all
2
3
4
5
Perfectly
55
Q

Primary Ciliary Dyskinesia

A

•  Autosomal recessive syndrome with variable penetrance
•  1 in 15,000 to 40,000 births, poorly functioning cilia
•  There is an absence or shortening of the dynein arms
•  Kartagener syndrome: bronchiectasis, sinusitus, and situs
inversus or partial lateralizing abnormality.

Therapy
•  Airway clearance •  Antibiotics •  Antinflammatory •  Preventing exacerbations

How well did you know this?
1
Not at all
2
3
4
5
Perfectly
56
Q

What is samter’s triad

A

Samter’s triad (Aspirin Exacerbated Respiratory Disease (AERD), also known as Samter’s Triad or Aspirin Sensitive Asthma)

  1. Asthma,
  2. Chronic rhinosinusitis with polyposis,
  3. aspirin/NSAID sensitivity)
How well did you know this?
1
Not at all
2
3
4
5
Perfectly
57
Q

Upper airway cough syndrome

A

Upper Airway Cough Syndrome
• Post nasal drip due to various etiologies
• Secretions in airway stimulate cough receptors in the laryngeal mucosa
• Common reason for chronic cough
• Tx: treat post nasal drip

How well did you know this?
1
Not at all
2
3
4
5
Perfectly
58
Q

Paradoxical Vocal Fold Motion/Vocal Cord Dysfunction (VCD)

A

Paradoxical Vocal Fold Motion/Vocal Cord Dysfunction (VCD)

*important to think for treatment resistant asthma
• Mistimed vocal fold closure creates difficulty breathing
– Vocal folds are held in a paramedian position through inspiration and
expiration
• Misdiagnosed as asthma
• Psychogenic component theorized
• Post nasal drip, inhaled or aspirated irritants, allergies or
GERD may cause laryngeal hyperresponsivenss
• TX- Treat underlying causes, reassurance, breathing
instruction

How well did you know this?
1
Not at all
2
3
4
5
Perfectly
59
Q

Vocal fold paralysis

unlateral vs bilateral

A

Vocal Fold Paralysis
• Unilateral- Usually asymptomatic
• Airway obstruction can be measured during both quiet and
active breathing
• The action of inspiratory airflow producing a Bernoulli effect
on the flaccid vocal fold (because they are now closer together, increases chance of the two collapsing on each other)
• Inappropriate reinnervation of the paralyzed vocal fold with
active signals for adduction during inspiration
• Tx- Surgery or botulinum toxin injection

Bilateral- Very symptomatic on exertion, weak, breathy voice
• Most commonly due to surgery in the anterior compartment of
the neck (Thyroid Surgery)
• Injury to recurrent laryngeal nerves
• Loss of vocal cord abduction during inspiration
• Loss of adduction for phonation
• Tx- Surgery, botulinum injection, tracheotomy

How well did you know this?
1
Not at all
2
3
4
5
Perfectly
60
Q

Glottic Stenosis

A

Glottic Stenosis

Scarring of the larynx • Usually posterior (PGS)
• Most common event associated with onset is prolonged
intubation (foreign body leading to excessive scarring)
• As the endotracheal tube rubs against the mucosa of the
posterior larynx, the mucosa is eroded and inflammation
develops
• Clinically and endoscopically it can be difficult to distinguish
from bilateral vocal fold paralysis
• PGS usually have a normal voice because vocal fold
adduction is maintained

How well did you know this?
1
Not at all
2
3
4
5
Perfectly
61
Q

Subglottic and Cervical Tracheal Stenosis

A

Subglottis is surrounded by a firm cartilaginous structure and
the mucosa lies over the surface with only a normal
submucosa for support of the epithelium
• Prone to injury from pressure or inflammation
• Injury to the mucosa by any one of the prior processes can
lead to exposure of the perichondrium, which then responds
with inflammation and scar tissue formation
• The scar tissue impedes airflow and mucus clearance

Caused by prolonged intubation, surgery, and autoimmune
disease (Granulomatosis with Polyangitis)
• Tx- Surgical

How well did you know this?
1
Not at all
2
3
4
5
Perfectly
62
Q

Significant predictors of hypersensitivity pneumonitis

ESPIRL

A

Exposure to known offending antigen - OR 38!

Symptoms 4-8 hours after exposure OR: 7

Positive precipitating antibodies

Inspiratory Crackles

Recurrent episodes of symptoms

weight Loss

Note: all six variables positive 98% chance of having HP; w/o weight loss 97% chance

How well did you know this?
1
Not at all
2
3
4
5
Perfectly
63
Q

croup

A

Caused by parainluenza viruses (paramyxovirus). Virus membrane contains hemagglutinin
(binds sialic acid and promotes viral entry) and neuraminidase (promotes progeny virion release)
antigens. Results in a “seal-like” barking cough and inspiratory stridor. Narrowing of upper trachea
and subglottis leads to characteristic steeple sign on x-ray

A . Severe croup can result in pulsus
paradoxus 2° to upper airway obstruction.

Respiratory illness characterized by a
hoarse voice, dry ‘barking’ cough,
inspiratory stridor and a variable amount of respiratory distress resulting from upper airway obstruction.

How well did you know this?
1
Not at all
2
3
4
5
Perfectly
64
Q

• 64 year old male admitted through the ER to the
ICU for chief complaints of fevers, cough and
shortness of breath. • Fever and rigors for 3 days. • Progressive shortness of breath for the last day • No significant past, family or occupational history • Current smoker (2 packs per day)
• Vital Signs: Temp 102 °F, pulse 130/min, BP 80/20mmHg, RR 28/m, room oxygen saturation by pulse oximetry: 89%. • Skin: Dry with reduced turgor • Cardiac exam: Tachycardia • Lung exam: Bronchial breath sounds, egophony, whispered pecteriloquy in the right lung base
posteriorly
• CBC: Hb: 14g%, Hct: 40, WBC: 20.000/μl, 85%
polymorphonuclear leucocytes, 14% bands, 1
lymphocyte. • Serum Chemistry: S.Na: 140, S.K: 4.0, S.Cl:
104, S.
H2Co3: 18, BUN: 72, S.Cr: 3.6, S.Glucose: 74 mg/dl, S. lactate: 6.0 mg/l
ABG: pH: 7.35, PaCO2 33 PaO2 60
Density in right lower lobe

A

he has consolidation
renal failure
lactic acidosis with hypoxia and huge A-a gradient.

Consolidation with right lower lobe

Severe Community Pneumonia with hypoxemia, leucocytosis, left shift, gap metabolic acidosis due to lactic acidosis.
• Blood and sputum culture, sensitivity was obtained.

How well did you know this?
1
Not at all
2
3
4
5
Perfectly
65
Q

What are the three kinds of ventilator induced lung injury

A

• Positive pressure ventilation results in
– End-inspiratory alveolar overdistention (volutrauma)
– End-expiratory alveolar derecruitment (atelectrauma)
– Biochemical injury and inflammation (biotrauma)

Volutrauma
• Large tidal volumes lead to increased wall stress (“stretch”)
– Gross physical disruption of lung tissue
– Activation of stretch-responsive inflammatory pathways in lung epithelial cells.
– Increased parenchymal inflammation, atelectasis, hypoxia and cytokine production.

Atelectrauma
• Cyclic closing and reopening of alveoli with each
tidal breath
• Alveolar shear-stress related injury
– Biochemical injury and inflammation (biotrauma) results from this mechanical stress

Lung Protection strategy
– Low tidal volumes (6cc/kg PBV) – Optimal PEEP – Recruitment maneuvers – Prone positioning

How well did you know this?
1
Not at all
2
3
4
5
Perfectly
66
Q

Rales vs Rhonchi

A

Rales = discontinuous sound, crackles, associated with alveoli, the most distal portion of airway
-usually due to fluid accumulation - pneumonia, pulmonary edema, can also be caused by atelectasis from PE

Rhonchi = wheezing, occurs in large airways, bronchus, continuous sound
-inflammatory in nature - asthma, chronic bronchitis, wheezing, constriction - or can be due to secretions

Rales in the tails, rhonchi in the bronchi!

How well did you know this?
1
Not at all
2
3
4
5
Perfectly
67
Q

What are vesicular breath sounds

Bronchial breath sounds

A

Vesicular

  • normal sound on most of lung
  • soft, low pitch
  • inspiration longer than expiration
  • no gap between phases

Bronchial
-abnormal in majority of lung that is far from main airways
-loud tubular quality
-high pitched
-inspiratory and expiratory phase equal
-definite gap between phases
HEARD in - Consolidation, lobar Collapse with patent bronchus, lung Cavity

How well did you know this?
1
Not at all
2
3
4
5
Perfectly
68
Q

What breath sounds are associated with consolidation?

Consolidation occurs through accumulation of inflammatory cellular exudate in the alveoli and adjoining ducts. Simply, it is defined as alveolar space that contains liquid instead of gas. The liquid can be pulmonary edema, inflammatory exudate, pus, inhaled water, or blood (from bronchial tree or hemorrhage from a pulmonary artery). It must be present to diagnose pneumonia: the signs of lobar pneumonia are characteristic and clinically referred to as consolidation

A
  1. increased tactile fremitus
  2. Bronchophony
  3. Aegophony (BEE heard as BAY)
  4. Whispering pectoriloquy
How well did you know this?
1
Not at all
2
3
4
5
Perfectly
69
Q

Wheeze breath sound

A

Continuous and musical quality

Expiratory usually

indicates narrowing of airways EITHER due to bronchospasm OR secretions in small airways

low pitch or high pitch

High pitch polyphonic or monophonic

during inspiration the elasticity of the airways will cause them to dilate which will allow air to flow around the obstruction. As the airways contract during exhalation, airflow will increase causing the high pitched sounds associated with wheezing

How well did you know this?
1
Not at all
2
3
4
5
Perfectly
70
Q

Crackles

A

Interrupted and non musical

inspiratory usually

peripheral airway collapse on expiration due to either interstitial fibrosis or secretions/fluid

During inspiration, rapid air entry abruptly opens these collapsed smaller airways and alveoli producing crackling noise.

Can be

  1. early inspiratory in small airways disease like broncholitis
  2. mid inspiratory in pulmonary edema
  3. Late inspiratory in pulmonary fibrosis, pulmonary edema, COPD, resolving pneumonia, lung abscess, tuberculous lung cavities
  4. biphasic in bronchiectasis

Fine crackles - broncholitis, pulmonary edema, pulmonary fibrosis

Coarse: COPD, resolving pneumonia, lung absess, tuberculous lung cavities, bronchiectasis

How well did you know this?
1
Not at all
2
3
4
5
Perfectly
71
Q

How to differentiate if a rub is pleural lining or pericardium,

A

you must perform a brief inspiratory hold maneuver, if the rub continues during the maneuver it is most likely a pericardial rub.

How well did you know this?
1
Not at all
2
3
4
5
Perfectly
72
Q

Lung cancer (general)

Presentation,
site of metastases from lung and to lung

SPHERE of complications:

Risk factors

Treatment

A

Leading cause of cancer death. Presentation: cough, hemoptysis, bronchial obstruction, wheezing, pneumonic “coin” lesion (solitary nodule) on CXR or non calcified nodule on CT.

  • Sites of metastases from lung cancer: adrenals, brain, bone (pathologic fracture), liver (jaundice, hepatomegaly).
  • In the lung, metastases (usually multiple lesions) are more common than 1° neoplasms. Most often from breast, colon, prostate, and bladder cancer.

SPHERE of complications:
Superior vena cava syndrome: obstruction of SVC leads to distended head and neck veins with edema and blue discoloration of arms and face (SVC syndrome)
Pancoast tumor: a tumor of the pulmonary apex. It is a type of lung cancer defined primarily by its location situated at the top end of either the right or left lung. It typically spreads to nearby tissues such as the ribs and vertebrae.
Horner syndrome
Endocrine (paraneoplastic)
Recurrent laryngeal nerve compression
(hoarseness)
Effusions (pleural or pericardial)
Risk factors include smoking, secondhand smoke,
radon, asbestos, family history.

Squamous and Small cell carcinomas are Sentral (central) and often caused by Smoking.

Small cell usually not amenable to surgical resection (chemotherapy and radiation)
Non-small cell: usually amenable to surgical resection, usually doesn’t respond well to chemo)

How well did you know this?
1
Not at all
2
3
4
5
Perfectly
73
Q
Small cell carcinoma 
Characteristic histology
Association
Location
Comment
A
[Small cell carcinoma]
Characteristic histology:
1. poorly differentiated small cells from neuroendocrine cells 
[chromogranin positive]
2. Neuron-specific enolase positive 

Association: male smokers
Location: central
Comment: rapid growth and early metastasis may produce endocrine (ADH or ACTH) or nervous system: (Labert-Eaton myasthenic syndrome: antibodies against presynaptic Ca2+ channels
Paraneoplastic myelitis, encephalitis, subacute cerebellar degeneration
Amplification of myc oncogenes common. Managed with chemotherapy +/- radiation

How well did you know this?
1
Not at all
2
3
4
5
Perfectly
74
Q
Adenocarcinoma (non small cell carcinoma)  
Characteristic histology
Association
Location
Comment 

+ bronchoalveolar subtype

A

[Adenocarcinoma]

Characteristic histology: glands or mucin production (stains positive)

Association: nonsmokers and female smokers (most common lung cancer overall excluding metastasis)

Location: Peripheral

Bronchoalveolar: grows along alveolar septa> apparent “thickening” of alveolar walls. Tall columnar cells containing mucus *may present as pneumonia like consolidation on imaging but excellent prognosis

Comment: activating mutations include KRAS, EGFR, and ALK. Associated with hypertrophic osteoarthropathy - clubbing
Bronchial carcinoid and bronchioalveolar cell have less association with smoking

How well did you know this?
1
Not at all
2
3
4
5
Perfectly
75
Q
Squamous cell carcinoma 
Characteristic histology
Association
Location
Comment
A

Squamous cell carcinoma

Characteristic histology: *keratin pearls OR intercellular bridges

Association: male smokers

Location: central

Comment:
Hilar mass arising from bronchus: 3C’s cavitation, cigarettes, hypercalcemia from PTHrP

How well did you know this?
1
Not at all
2
3
4
5
Perfectly
76
Q
Large cell carcinoma
Characteristic histology
Association
Location
Comment
A

Characteristic histology: pleomorphic giant cells

Association: smoking

Location: central or peripheral

Comment : highly anaplastic undifferentiated tumor, POOR prognosis, less responsive to chemotherapy, removed surgically. Strong association with smoking .

How well did you know this?
1
Not at all
2
3
4
5
Perfectly
77
Q

Bronchial carcinoid tumor

A

Excellent prognosis, metastasis rare

SYmptoms due to mass effect or carcinoid syndrome: flushing, diarrhea, wheezing

Nests of neuroendocrine cells, chromogranin A positive like small cell carcinoma.

Not related to smoking

central or peripheral, classically presents as a polyp-like mass in the bronchus. Its a low grade malignancy!

How well did you know this?
1
Not at all
2
3
4
5
Perfectly
78
Q

Common causes of pneumonia

Neonates (<4 wk) 
Children (4 wk - 18 years) 
Adults (18-40) 
Adults (40-65)
Elderly
A

Neonates (<4 wk)
Group B streptococci
E coli

Children (4 wk - 18 years) 
-Viruses (RSV) 
-Mycoplasma
-C trachomatis (infant s –3 y r) 
-C pneumoniae (scho ol- a ged
children)
S pneumoniae
Runts May Cough
Chunky Sputum
Adults (18-40) 
Mycoplasma 
C pneumoniae 
S pneumoniae 
Viruses (eg, influenza
Adults (40-65)
S pneumoniae 
*H inluenzae 
*Anaerobes 
Viruses 
Mycoplasma
Elderly (65+)
S pneumonia 
Influenza virus 
Anaerobes
H influenza
Gram negative rods
How well did you know this?
1
Not at all
2
3
4
5
Perfectly
79
Q

Pathoma:

What are the high yield types of lung benign lesions and their presentation

A

coin lesion especially in younger patients (<40)

  1. Granuloma- often due to TB or fungus
    * *If Midwest - Histoplasma
  2. bronchial hamartoma - benign tumor composed of lung tissue and cartilage often calcified on lung
How well did you know this?
1
Not at all
2
3
4
5
Perfectly
80
Q

If you find a solitary nodule on xray, what is the next step?

A

Look at old xrays

How well did you know this?
1
Not at all
2
3
4
5
Perfectly
81
Q

What is TNM staging system (Stage - spread)

pulm

A

TNM staging system (Stage = Spread):
T = Tumor size/invasiveness - obstruction of SVC leads to distended head and neck veins with edema and blue discoloration of arms and face (SVC syndrome)
-involvement of recurrent laryngal (hoarseness or phrenic ( diaphragmatic paralysis nerve)
-involvement of the SNS chain (Horners) and brachial plexus (should pain and hand weakness is seen with pancoast tumors!

N = Node involvement
-hilar, mediastinal

M = Metastases Each TNM factor has independent prognostic
value; N and M factors are often most
important.
**adrenal gland is high yield

How well did you know this?
1
Not at all
2
3
4
5
Perfectly
82
Q

exudate vs transudate

Light criteria

A
[exudate]
Cellular (cloudy) 
elevated protein (> 2.9 g/dL) 
elevated LDH (vs serum) 
Due to:
ƒ Lymphatic obstruction (chylous)
ƒ Inlammation/infection
ƒ Malignancy 

[transudate)
Hypocellular (clear)
low protein (< 2.5 g/dL)
low LDH (vs serum)
Due to:
ƒ increased hydrostatic pressure (eg, HF, Na+ retention)
ƒreduced oncotic pressure (eg, cirrhosis, nephrotic syndrome

Light Criteria
Diagnostic analysis comparing serum and pleural fluid protein and LDH levels. Pleural effusion is exudative if ≥ 1 of the following criteria is met:
ƒ Pleural effusion protein/serum protein ratio > 0.5
ƒ Pleural effusion LDH/serum LDH ratio > 0.6
ƒ Pleural effusion LDH > 2⁄3 of the upper limit of normal for serum LDH

How well did you know this?
1
Not at all
2
3
4
5
Perfectly
83
Q

Restrictive lung diseases

A

Restricted lung expansion causes reduced lung volumes (reduced FVC and TLC).
PFTs: FEV1/FVC ratio ≥ 80%.
Patient presents with short, shallow breaths.
Ty p e s :
1. Poor breathing mechanics (extrapulmonary, peripheral hypoventilation, normal A-a gradient):
ƒ-Poor muscular effort—polio, myasthenia gravis, Guillain-Barré syndrome ƒ
-Poor structural apparatus—scoliosis, morbid obesity
ƒ2. Interstitial lung diseases (pulmonary reduced diffusing capacity, increased A-a gradient):
ƒ-Pneumoconioses (eg, coal workers’ pneumoconiosis, silicosis, asbestosis) ƒ

-Sarcoidosis: bilateral hilar lymphadenopathy, noncaseating granuloma; increased ACE and Ca2+ ƒ

-Idiopathic pulmonary fibrosis (repeated cycles of lung injury and wound healing with
increasedcollagen deposition, “honeycomb” lung appearance and digital clubbing) ƒ
• Repeated cycles of epithelial activation/injury by some unidentified agent followed by abnormal
epithelial repair at the site of injury
• Leads to excessive fibroblastic proliferation resulting in characteristic fibroblastic foci seen on
lung biopsy
• Areas of both early injury/repair and late collagen deposition lead to characteristic “temporal
heterogeneity”
Radiographic feature: basal-predominant reticular abnormality with volume loss.

Granulomatosis with polyangiitis: inflammation of multiple blood vessels or lymph vessels (Wegener) ƒ

Pulmonary Langerhans cell histiocytosis (eosinophilic granuloma) ƒ

Drug induced pulmonary fibrosis: (bleomycin, busulfan, amiodarone, methotrexate)

Hypersensitivity pneumonitis—mixed type III/IV hypersensitivity reaction to environmental antigen. Causes dyspnea, cough, chest tightness, headache. Often seen in farmers and those exposed to birds. Reversible in early stages if stimulus is avoided.
• Also called extrinsic allergic alveolitis • Results from repeated inhalation of finely dispersed
antigens
• Antigens encompass a wide variety of organic
particles
• Sources include mammalian and avian proteins,
thermophilic bacteria, fungi, certain small MW chemical compounds

• Acute form results from intermittent and intense
exposure • Symptoms occur 4-8 hrs. after exposure and
characterized by fever, chills, malaise, dyspnea and
cough

• Subacute and chronic forms result from continual, low-level exposure, usually in the domestic environment • Onset insidious; main symptoms are dyspnea and
fatigue • Unrecognized and untreated subacute form may
progress to chronic form with irreversible lung changes • Exam usually shows tachypnea and crackles in all forms

• Acute form shows diffuse ground-glass appearance
or air-space consolidation

• In subacute form CXR shows fine nodular or reticulonodular pattern

• Chronic form shows predominantly a reticular
pattern • Distribution involves mainly the upper two-thirds
of the lungs

How well did you know this?
1
Not at all
2
3
4
5
Perfectly
84
Q

Inhalation injury and sequelae

A

Pulmonary complication associated with smoke
and fire.
Caused by heat, particulates ( chemical
tracheobronchitis, edema, pneumonia,
ARDS. Many patients present 2° to burns, COinhalation, cyanide poisoning, or arsenic
poisoning.

Bronchoscopy shows severe edema, congestion of bronchus, and soot deposition ( A , 18 hours
after inhalation injury; B , resolution at 11 days after injury).

How well did you know this?
1
Not at all
2
3
4
5
Perfectly
85
Q

Pneumoconiosis

A

Coal workers’ pneumoconiosis, silicosis, and asbestosis > increased risk of cor pulmonale, cancer, and Caplan syndrome (rheumatoid arthritis
and pneumoconioses with intrapulmonary nodules).

Asbestos is from the roof (was common in insulation), but affects the base (lower lobes). Silica and coal are from the base (earth), but
affect the roof (upper lobes).

How well did you know this?
1
Not at all
2
3
4
5
Perfectly
86
Q

Asbestosis

A

Associated with shipbuilding, rooing,
plumbing. “Ivory white,” calcified, supradiaphragmatic and pleural plaques
are pathognomonic of asbestosis. Risk of bronchogenic carcinoma > risk of
mesothelioma.

Affects lower lobes. Asbestos (ferruginous) bodies are golden-brown
fusiform rods resembling dumbbells,
found in alveolar sputum sample, visualized using Prussian blue stain, often obtained by
bronchoalveolar lavage.
increased risk of pleural effusions (consistent with increased risk of lung cancer)

How well did you know this?
1
Not at all
2
3
4
5
Perfectly
87
Q

Berylliosis

A

Associated with exposure to beryllium in
aerospace and manufacturing industries.
Granulomatous (noncaseating) on histology
and therefore occasionally responsive to steroids.

Affects upper lobes

How well did you know this?
1
Not at all
2
3
4
5
Perfectly
88
Q

Coal workers pneumoconiosis

A

Prolonged coal dust exposure > macrophages
laden with carbon > inflammation and
fibrosis. Also known as black lung disease.

Affects upper lobes.
Anthracosis—asymptomatic condition found in many urban dwellers exposed to sooty air.

89
Q

Silicosis

A

*most common worldwide

Simple vs complicated silicosis

Simple: mildest, earliest form of disease
-asymptomatic or have chronic cough
CXR shows upper zone distribution of small rounded opacities less than 10 mm in diameter • Symptoms do not correlate with CXR findings • PFT’s initially normal but TLC decreases as the number/density of opacities increase • Tuberculosis incidence increased by 2-30 fold

Associated with foundries, sandblasting,
mines. Macrophages respond to silica
and release fibrogenic factors, leading to fibrosis. It is thought that silica may disrupt phagolysosomes and impair macrophages, increasing susceptibility to *****TB.

Affects upper lobes
Eggshell calcification in hilar lymph nodes on CXR - nicely surrounded

Complicated Silicosis
• AKA: Progressive Massive Fibrosis • Occurs when smaller opacities coalesce • Symptoms range from minimal to severe dyspnea • CXR shows confluent nodules > 10mm • Nodules become confluent peripherally and migrate centrally • PFT’s show progressive deterioration • Progression in absence of further exposure

90
Q

Mesothelioma

A

Malignancy of the pleura associated with
asbestosis. May result in hemorrhagic pleural
effusion (exudative), pleural thickening.
Psammoma bodies seen on histology. Cytokeratin and calretinin ⊕ in almost all mesotheliomas, ⊝ in most carcinomas.

Smoking not a risk factor.

91
Q

Acute respiratory distress syndrome

A

Diagnosis of exclusion characterized by respiratory failure within 1 week of alveolar insult, bilateral lung opacities, decreased PaO2/FiO2 < 300 (hypoxemia due to increasedintrapulmonary
shunting and diffusion abnormalities),
no evidence of HF/fluid overload. Many
causes and associations, including sepsis,
*pancreatitis, pneumonia, aspiration, trauma,
shock.
Endothelial damage leads to increasedalveolar
capillary permeability > protein-rich leakage into alveoli > diffuse alveolar damage and noncardiogenic pulmonary edema (normal
PCWP)
Results in formation of intra-
alveolar hyaline membranes
Initial damage due to release of neutrophilic substances
toxic to alveolar wall and pulmonary capillary
endothelial cells, activation of coagulation
cascade, and oxygen-derived free radicals.

Management: mechanical ventilation with low
tidal volumes, address underlying cause.

92
Q

Sleep apnea

Obstructive
Central
Obesity hypoventilation syndrome

A

Sleep apnea:
Repeated cessation of breathing > 10 seconds during sleep > disrupted sleep leads to increased daytime somnolence. Diagnosis confirmed by sleep study. Normal Pao2 during the day. Nocturnal hypoxia > systemic/pulmonary hypertension, arrhythmias (atrial fibrillation/flutter), sudden death. Hypoxia > increased EPO release > increased erythropoiesis.

Obstructive:
Respiratory effort against airway obstruction. Associated with obesity, loud snoring. Caused by
excess parapharyngeal tissue in adults, adenotonsillar hypertrophy in children. Treatment: weight loss, CPAP, surgery.

Central sleep apnea:
No respiratory effort due to CNS injury/toxicity, HF, opioids. “Central sleep apnea is a disorder in which your breathing repeatedly stops and starts during sleep. Central sleep apnea occurs because your brain doesn’t send proper signals to the muscles that control your breathing. … Central sleep apnea is less common than obstructive sleep apnea”

May be associated with Cheyne-Stokes respiration.
Treat with positive airway pressure.

Obesity hypoventilation syndrome:
Obesity (BMI ≥ 30 kg/m2) > hypoventilation > increased PaCO2 during waking hours (retention); decreasedPaO2
and increased PaCO2 during sleep. Also known as Pickwickian syndrome.

93
Q

Crackles (lecture associatons)

A
  1. Idiopathic pulmonary fibrosis (usual interstitial pneumonia)
  2. Hypersensitivity pneumonitis (inspiratory)
  3. Asbestosis (basal crackles)
  4. acute pulmonary embolism
94
Q

Sarcoidosis

A

Characterized by immune-mediated, widespread noncaseating granulomas , elevated serum
ACE levels, and elevated CD4+/CD8+ ratio in bronchoalveolar lavage fluid.
More common in African-American females. Often asymptomatic except for enlarged lymph nodes. Findings on CXR of bilateral adenopathy and coarse reticular opacities; CT of the chest better demonstrates
the extensive hilar and mediastinal adenopathy - bilateral hilar is most common
C . Associated with *restrictive lung disease (interstitial fibrosis), erythema nodosum, lupus pernio (skin lesions on face resembling lupus), Bell palsy, epithelioid granulomas containing microscopic
Schaumann and asteroid bodies, uveitis, hypercalcemia (due to q 1α-hydroxylase–mediated vitamin D activation in macrophages).

Treatment: steroids (if symptomatic).

Lecture: extrapulmonary sarcoidosis
• Ophthalmic granulomatosis • Peripheral adenopathy • Cutaneous rash • Central nervous system involvement • Cardiac arrhythmias • Hepatitis • *Hypercalcemia: one of the first symptoms

95
Q

Granulomatosis with polyangiitis (Wegener’s)

A

c-ANCA associated vasculitis which can cause cavitary lesions in the lungs. most patients are oldr males with chronic sinusitis, hemoptysis, and renal involvement. THese patients may have low grade fever and lung cultures would be negative.

Granulomatosis with polyangiitis (GPA), also known as Wegener’s granulomatosis (WG), is a systemic disorder that involves both granulomatosis and polyangiitis. It is a form of vasculitis (inflammation of blood vessels) that affects small- and medium-size vessels in many organs. Damage to the lungs and kidneys can be fatal. Treatment requires long-term immunosuppression.[1]

Granulomatosis with polyangiitis is part of a larger group of vasculitic syndromes called systemic vasculitides or necrotizing vasculopathies, all of which feature an autoimmune attack by an abnormal type of circulating antibody termed ANCAs (antineutrophil cytoplasmic antibodies) against small and medium-size blood vessels. Apart from GPA, this category includes eosinophilic granulomatosis with polyangiitis (EGPA) and microscopic polyangiitis.[1] Although GPA affects small- and medium-size vessels,[2] it is formally classified as one of the small vessel vasculitides in the Chapel Hill system

Can cause subglottic and tracheal cervical stenosis

96
Q

Hereditary thrombosis syndromes leading to hypercoagulability

Antithrombin
Factor V Leiden
Protein C or S deficiency
Prothrombin gene mutation

A
  1. Antithrombin
    deficiency:
    Inherited deiciency of antithrombin: has no direct effect on the PT, PTT, or thrombin time but
    diminishes the increase in PTT following heparin administration. Can also be acquired: renal failure/nephrotic syndrome > antithrombin loss in urine
    >reducedinhibition of factors IIa and Xa.
  2. Factor V Leiden
    Production of mutant factor V (G pA DNA point mutation > Arg506Gln mutation near the cleavage site) that is resistant to degradation by activated protein C. Most common cause
    of inherited hypercoagulability in Caucasians. Complications include DVT, cerebral vein thromboses, recurrent pregnancy loss.
  3. Protein C or S
    Reduced ability to inactivate factors Va and VIIIa. Increased risk of thrombotic skin necrosis with hemorrhage
    after administration of warfarin. If this occurs, think protein C deiciency. Together, protein C
    Cancels, and protein S Stops, coagulation.
  4. Prothrombin gene mutation: Mutation in 3′ untranslated region > increased production of prothrombin > increased plasma levels and venous
    clots.
97
Q

DVT

A

Blood clot within a deep vein > swelling,
redness , warmth, pain. Predisposed by
Virchow triad (SHE):
ƒStasis (eg, post-op, long drive/light)
Hypercoagulability (eg, defect in coagulation cascade proteins, such as factorV Leiden)
ƒEndothelial damage (exposed collagen
triggers clotting cascade) d-dimer lab test used clinically to rule out DVT
(high sensitivity, low speciicity).

Most pulmonary emboli arise from proximal deep veins of lower extremity.
Use unfractionated heparin or low-molecular-weight heparins (eg, enoxaparin) for
prophylaxis and acute management. Use oral anticoagulants (eg, warfarin,
rivaroxaban) for treatment (long-term
prevention).
Imaging test of choice is compression ultrasound with Doppler.

98
Q

Rhinosinusitis

A

Obstruction of sinus drainage into nasal cavity > inflammation and pain over affected area
(typically maxillary sinuses, filled with fluid on the right in, which drain into the middle meatus, in adults). Most common acute cause is viral URI; may cause superimposed bacterial infection, most commonly
1. S pneumoniae,
2. H inluenzae,
3. Mcatarrhalis.

99
Q

Compare venous and arterial thrombi

A

Arterial

  • form under high shear stress
  • endothelial cell is disrupted
  • thrombus is platelet rich
  • use anti-platelet drugs
  • “white clots”
Venous 
-form under low shear stress
-endothelial cell is intact
-fibrin-rich 
-use anti-coagulants for treatment
“red clots”
100
Q

Paget-Shroetter Syndrome

May-Thurner syndrome

A

Paget-Shroetter Syndrome:

  • spontaneous upper extremity thrombosis
  • compression at the thoracic outlet
  • # 1 cause of SPONTaneous UE DVT

May-Thurner syndrome
-compression of the left common iliac vein by the right common iliac artery

101
Q

Phlegmasia Cerulae Dolens

A

Massive proximal DVT of the lower extremities

Signs and symptoms include:

  • sudden severe leg pain with swelling
  • cyanosis
  • edema
  • various gangrene
  • arterial compromise
102
Q

increased Exhaled nitric oxide (FeNO) is a marker for what

A

a marker of Th2

eosinophil mediated inflammation

103
Q

M3 receptor stimulation [pulm)

A

Parasympathetic nerve stimulation produces
bronchoconstriction, mediated by M3 receptors.

• M3 receptor activation also increases airway secretions

104
Q

allergic bronchopulmonary aspergillosis

A

A cause of necrotizing inflammation to bronchioles and bronchi, causing bronchiectasis

-hypersensitivity reaction to aspergillosis leading to chronic inflammatory damage. Usually seen in individuals with asthma or cystic fibrosis

105
Q

HIV associated pulmonary disease

CD4+ count

Any 3
<200 cells/uL 4
<100 cells/uL 4
50-100 cells/uL 4

A

Any:

  1. bacterial pneumonia
  2. TB
  3. non-Hodgkin lymphoma

<200 cells/uL:

  1. Bacterial pneumonia w/ bacteremia
  2. disseminated TB
  3. Pneumocystis TB
  4. Cryptococcus neoformans

<100 cells/uL:

  1. Staphylococcus
  2. Pseudomonas
  3. Kaposi sarcoma (pulm manifestations)
  4. Toxoplasmosis

50-100 cells/uL:

  1. Endemic fungi
  2. Mycobacterium avium complex (MAC)
  3. nonedemic fungi
  4. CMV
106
Q

•  A 29-year-old woman with HIV infection and a CD4 cell
count of 633/µL has had 3 days of fever, chills,
productive cough, and chest pain. Physical examination
shows signs of consolidation in the left lower lung fields.
Her leukocyte count is 8600/µL, and chest radiograph
shows a left lower lobe infiltrate. •  Which of the following organisms is most likely
present in her sputum? • 
( A ) Mycoplasma pneumoniae
( B ) Streptococcus pneumoniae
( C ) Legionella pneumophila
( D ) Pseudomonas aeruginosa
( E ) Pneumocystis carinii

A

(B) streptococcus pneumonia

CD4 is high enough

107
Q

A Chinese immigrant has a negative Inteferon Gamma
Release Assay but a positive PPD? What is the best
implication?
–  A) Prior infection from Mycobacterium TB – isolate or refer to the
immigration policy of the current President – 
B) Prior vaccination via BCG – 
C) Active TB – 
D) Recent TB exposure – 
E) Treated disease (from TB)

A

B) prior vaccination via BCG

PPD tells you nothing about the time course.

108
Q

38yf, from Korea 6-months ago. WL, cough, hemoptysis for 4-months. No fever. CXR 4cm RUL cavitary. BCG scar (16yr). PPD 18mm. 5-
sputum samples – negative. Cultures pending. Bronch nothing. At this point in the workup what is false?
•  A) Negative smears for mycobacteria do not exclude TB, but
the diagnosis is less likely.
•  B) Despite BCG vaccination, the reaction of 18mm most likely
represents Myco TB infection
•  C) Sputum should assessed for other etiologies, it’s not TB
•  D) Patient should be started upon RIPE until culture results
•  E) Culture negative, but clinical response occurs to RIPE, the
patient has “culture negative TB” and be treated for 4-months
total

A

Sputum should assessed for other etiologies not Tb

109
Q

Lobar pneumonia

cause + four stages

A
•  Seen in bacterial pneumonia
•  Four stages of inflammatory
response:
–  Congestion 
–  Red hepatization 
–  Gray hepatization 
–  Resolution
•  (But might have residual fibrous
thickening, adhesions, scarring)
110
Q

Streptococcus pneumoniae

A

G+, alpha hemolytic (surrounding zone is a green hue)
-optochin sensitive, lancet shaped diplococci, bile soluble

**Most Common Cause of Community acquired pneumonia

•  Sputum -- rust colored, lobar pneumonia 
MOPS; meningitis, otitis media, pneumonia, sinusitis  #1 cause of all of these. 
•  Alveolar architecture
preserved
–  When minimal residual
destruction
•  If baseline compromised
lung function -- can lead
to decompensation 
•  20% of adults – normal flora
–  Need clinical and
infectious picture too

Some serotypes can cause empyema and bronchopleural fistula.

Treatment: azithromycin, macrolides, ceftriaxone

111
Q

Haemophilus influenzae

A

Haemophilus Influenzae
•  Most common cause of of bacterial acute COPD
exacerbation

•  A-F serotypes, pleomorphic G negative
–  Vaccine is against b – most virulent form
•  Capsulated forms •  Non-capsulated forms emerging more
–  Incidence increasing, some non-
typeable –  URI symptoms first – otitis media, sinusitis, bronchopneumonia
•  Neonates, cancer patients, Immune
compromised
•  Pediatric Emergency – high mortality
rate pre-vaccine

112
Q

Moraxella catarrhalis

A

Moraxella catarrhalis
•  Seen more in elderly, Gram negative diplococus •  2nd Most common Cause of bacterial COPD exacerbation
•  Common cause of otitis media in children
•  Another bronchopulmonary pathologic agent
•  Perhaps 30% of humans are colonized upper respiratory tract
early

113
Q

Staph aureus (pulm)

A

Staphylococcus Aureus
•  MSSA – in the community, but MRSA-CAP can be seen too
–  Hospital acquired can be MRSA more
•  Common cause of 2ndary bacterial pneumonia after virus/influenza
•  Can lead to persistent anatomic
complications abscess/empyema/cavitations
•  IV Drug Users – seed the lung via bacteremia first
•  Daptomycin inactivated by surfactant – so not great for PNA

Vancomycin

114
Q

Klebsiella pneumoniae

A

Klebsiella Pneumoniae
•  Most common cause of Gm negative bacterial pneumonia
•  Debilitated, malnourished, alcoholics
•  Thick, mucoid, blood tinged sputum
–  Produces viscid capsular polysaccharide which makes it tough to expectorate
•  Can form abscess

3A’s - alcoholics, abscesses, aspiration
-Klebsiella is immotile
-cavatary lesion on patients right lobe is tb like
urease positive
treat with ceftriaxone, cefotaxime

115
Q

Legionella pneumophila

A

Legionella pneumophila

•  Thin, pleomorphic, flagellated, Gram negative
•  Artificial aquatic environments
–  Water cooling and tubing systems
•  Again, common in sicker people and smokers
•  Rapid dx
–  Legionella Urine Antigen (only tests for a subtype)
–  Culture is gold standard though (pathogenic regardless of how much •  Can be fatal in up to 15% •  Variable radiographic appearnce
–  Infiltrates or consolidation •  Need to identify since it’s outbreak associated •  Atypical
–TX is  Fluorquinolone or macrolide

visualized under silver stain - need cysteine and iron to be added to the agar.
Pontiac fever - fever and malaise usually is self limited.
Common in smokers and elderly men

Blue print: atypical pneumonia patchy unilobed infiltrate

(FA) Legionnaires’ disease—severe pneumonia
(often unilateral and lobar, fever, GI and
CNS symptoms. Common in smokers and in chronic lung disease.
Pontiac fever—mild flu-like syndrome.

116
Q

Mycoplasma

A
Mycoplasma
•  Another atypical. 
1/3 of Outpatient CAP
–  10d for culture growth 
•  Mollicutes – bacterium without
cell wall 
•  “Walking pneumonia” 
•  Can have 
•  Close quarters 
•  Pathologic if isolated 
•  Dx via PCR or Serology
 •  Not sensitive to Beta-Lactams
–  Fluorquinolone or macrolide (z pack)

no stain color - grain intermediate
cholesterol in the cell membrane, sterols in the membrane
-atypical pneumonia because can’t readily culture, walking pneumonia
-patchy infiltrate.

117
Q

Hyperacute Sx -  Sepsis

-  No URI Sx -  URI then get much worse -  WBC 15+ or 6- -  Dense lobar consolidation High Procalcitonin

A

Bacterial
or
Legionella pneumonia

118
Q

-Absence of bacterial factors -Family Cluster -Cough 5d+ no acute deterioration -No sputum -WBC nml or slight up -Low procalcitonin

A

Atypical bacterial (mycoplasma)

119
Q

-Absent bacterial factors -Sick contacts -URI concurrent -Patchy infiltrates -WBC nml or slight up -Low procalcitonin

A

viral pneumonia

120
Q

-Absent bacterial factors -CDC reporting in community -Sudden onset flu sx (malaise, myalgia, fatigue, lethargy

A

Influenza pneumonia

121
Q

Pseudomonas aureginosa

A
Pseudomonas Aureginosa
•  Gm (-) rod 
•  You’ll likely be colonized by it
•  Lung disease, CF (most common gram negative nosocomial pneumonia, respiratory failure in CF patients, bronchiectasis
•  Immune suppressed
•  Usually easy to culture
•  Double coverage
–  B-lactam/FLQN resistance
–  Advanced agents

Thrives in aquatic environments, hot tub folliculitis

Blue ring - oxidase positive and catalase positive

Obligate aerobe

Causes otitis externa, ecthyma gangrenosom (dalmation)

Treatment: piperacillin, aminoglycosides and fluoroquinolones

122
Q

Histoplasmosis

A

\
•  In the US, more in the Mississippi River Areas
•  Inhalation of dust particles from soil contaminated with bird or
bat droppings that contain infectious spores
•  Clinical
–  (1) Self limited or latent pulmonary involvement – “coin lesions”
–  (2) Then can go to….chronic, progressive secondary lung disease
•  Lung apices, cough, fever night sweats –  (3) Spread to extrapulmonary sites –  (4) Wide dissemination (especially in the Immune Compromised)

•  Target Macrophages
–  Can opsonize into them - macrophages with intracellular oval bodies
–  Multiply and lyse
•  Helper T cells usually see fungal cell wall antigens
–  Interferon gamma secreted and activates macrophages and kill IC yeasts
•  Also Histoplasma induces macrophages to secrete TNF which
recruits and stimulates other macrophages to kill Histoplasma
–  NOTE RISK FACTORS FOR PEOPLE ON TNF INHIBITORS!
•  Diagnosis by culture or Identification in tissue lesions •  Serologic Antibodies
–  2-6wk after infection

•  Normal hosts recover 99% unless infective dose high and get entire lung involved
•  *Very close clinically to TB (book w/ Tb symbol)
•  Very similar to sarcoidosis
–  That is why in treating sarcoid, you
want to exclude fungal disease before beginning Immune Suppression
•  Symptoms are usually anatomically based
and self limited
•  Disseminated forms in impaired T-Cell suppression (can’t form granulomas)
-Granulomas cause compression (granulomas mirror RB, w calcified nodes and nodules in the hilar region)
-Broncholithiasis (LN erodes into trachea)
-Fibrosing Mediastinitus (exstensive fibrosis of mediastinum from large antigen release – can get Supperior Vena Ceva blockage, airway constriction)
-Cavitary Histoplasmosis (in patients which abnormal lungs

mold in the cold and yeast in the beast
diagnosis by KOH or rapid serum or urine antigen test

123
Q

Blastomycosis

A
Blastomycosis
•  Blastomyces dermatitidis is a soil-inhabiting dimorphic fungus. 
•  Central and southeastern United States  **great lakes and Ohio river valley
•  Forms
–  pulmonary blastomycosis
•  Usually abrupt onset illness •  Vary radiographic presentation (lobar, diffuse) 
•  Usually resolves spontaneously
–  disseminated blastomycosis
–  (rare) primary cutaneous form
•  Diagnosis
–  Antigen in urine or serum
–  BAL shows “Broad-Based Budding yeast”

Treatment is in the immunocompetent
-litraconazole, systemic infection use amphotericin B
Mold in the cold and yeast in the heat -dimorphism.
-blasto is typically the same size as RBCs = patchy alveolar infiltrate

124
Q

Coccidiomycosis

A

Coccidiomycosis
•  *Southwestern US, Very pathogenic – *inhalation usually leads to infection
–  The fungus seems to block lysosome
•  Delayed type hypersensitivity reaction phagosome fusion
•  Peripheral eosinophilia common
•  *Still usually asympomatic
–  10% progress to lung lesions, cough, pleuritic pain and erythema nodosum or erythema multiforme (San Joaquin Valley fever complex), knee pain - travels to one
–  1% go to disseminated disease

Mold in the cold *spherule of endospores in the body, not yeast.

Azoles for local, amphotericin B for systemic

125
Q

Cryptococcus

A

Cryptococcus
•  Budding HEAVily encapsulated
yeast also ubiquitous in
soil •  HIV association •  India Ink stain

transmitted by pigeon droppings and found in the soul.
Urease positive
Spread to CSF and cause meningitis, this is no bueno and very often will lead to permanent neuro deficits

Treatment with ampho B, flucytosine and then fluconazole

126
Q

Murcormycosis

A
Mucormycosis
•  Treatment for
Aspergillus can be risk
factor 
•  Can first infect sinuses 
•  More nodular than
aspergillus Necrotic,
Cavitation 
•  90-degree branching

transmitted via spore inhalation - diabetic ketone acidosis predisposes infection of this fungus.
Invade through cribriform plate in the skull then will continue to cause necrosis of tissue and frontal cortex abscesses
-black eschar and necrosis of nasal cavity and eyes, causing neuro deficits and death

Amphotericin B

127
Q

Candida

A

Opportunistic Mycoses – Candida
•  Normal flora in GI and GU tracts, and commonly contaminates cultures
-severe diaper rash when exposed to heat and humidity.
•  Can be invasive and cause pneumonia though
•  Pneumonia from just candida is extremely rare
–  Usually from bloodstream infections (not oropharyngeal secretions)
–  Can see micro-absscesss and lobar infiltrates
•  Rarely treat when isolated in sputum or at bronchoscopy
•  More when see signs of disseminated infection

oral candida common in asthmatic kids that don’t rinse mouth after use of inhaler
-esophagitis
Aids defining illness at CD4 of 100

128
Q

Pneumocystis pneumonia

A

Pneumocystis pneumonia
•  Seen in HIV and Immune Suppressed patients
–  Immune Suppressed (CXR to right) has more neutrophils on bronchoscopy, fewer
cysts on metheneamine silver stains
•  Seen more as fungi than protozoa •  Prophylaxis has reduced incidence •  Can be spread airborne – person to person
–  Normal immune system individuals can be colonized with it
•  Can present fulminant or subacute
– dry cough and fever
–  HIV patients usually more subacute and bronchoscopy fluid has more lymphocytes
and (CD8+) cells, not many neutrophils, more cysts (below) –  HIV candidiasis can predispose
•  Tend to see desaturation on exertion

Pneumocystis jiroveci – PCP Ping Pong
1. Aid for Aids – Associated with Aids CD4 counts below 200
2. 20-0 – CD4 counts below 200
3. Immunocompromised Cane player and young player – Symptoms are evident in immunocompromised individuals
4. Cracked glass ping pong tables - Will have a ground glass appearance in both lungs
5. BAL water bottle - Broncheolavar lavage for diagnosis
6. Silver discs on the table and ovoid ping pong balls - Methamine silver stain to identify fungus
that looks like disc shaped yeasts
7. Backhand, and the jar of ping pong sulfa bottle- Prophylaxis begins when CD4 count is below
200, Bactrim (TMP/SMX)
8. Pentagon paddles – Pentadamine can be used with sulfa allergies

129
Q

Cytomegalovirus

A

Cytomegalovirus
•  Immune Suppressed unique population – Transplant
–  Get from CMV+ donor
–  Blood products from CMV+ donor
–  Reactivation of latent infection in seropositive recipient
–  Close contact with CMV infected individual
•  “CMV Infection” – evidence of CMV regardless of symptoms
or sings
•  “CMV disease” – signs and symptoms
–  Fever, malaise (vial syndrome) –  Tissue invasive disease (pneumonitis)

130
Q

What are the five causes of hypoxemia?

A
  1. Low FiO2 (house fire or high altitude) - alveolar oxygen tension is low
  2. hypoventilation - alveolar oxygen tension is low

A-a gradient would be normal for alveolar oxygen tension.

  1. V/Q mismatch high or low (most common cause of hypoxemia)
  2. Shunt
  3. Diffusion impairment - interstitial lung disease and that’s IT

A-a gradient is high

131
Q

Common causes of pneumonia

A

NEONATES (< 4 WK)

  1. Group B streptococci
  2. E coli
CHILDREN (4 WK–18 YR)
Viruses (RSV) Mycoplasma
C trachomatis (infant s –3 y r) 
C pneumoniae (school aged
children)
S pneumoniae
Runts May Cough
Chunky Sputum
ADULTS (18–40 YR)
Mycoplasma 
C pneumoniae 
S pneumoniae 
Viruses (eg, inluenza)
ADULTS (40–65 YR)
S pneumoniae 
H influenzae 
Anaerobes 
Viruses 
Mycoplasma
ELDERLY
S pneumoniae 
Influenza virus 
Anaerobes 
H inluenzae 
Gram ⊝ rods
132
Q
Common causes of pneumonia in 
alcoholics
IV drug user
Aspiration
Atypical
Cystic fibrosis
Immunocompromised
Nosocomial 
Postviral
A
  1. Alcoholic: Klebsiella, anaerobes usually due to aspiration (eg, Peptostreptococcus, Fusobacterium, Prevotella, Bacteroides)
  2. IV drug users: S pneumoniae, S aureus
  3. Aspiration: Anaerobes
  4. Atypical: Mycoplasma, Legionella, Chlamydia
  5. Cystic fibrosis: Pseudomonas, S aureus, S pneumoniae, Burkholderia cepacia
  6. Immunocompromised: S aureus, Pseudomonas, other enteric gram ⊝ rods, fungi, viruses, P jirovecii with HIV
  7. Nosocomial- S. aureus, Pseudomonas, other enteric gram negative rods
  8. Postviral: S pneumoniae, S aureus, H influenza.
133
Q

Most common cause of community acquired pneumonia

A

Streptococcus pneumoniae (90%)

134
Q

Most common bacterial cause of acute exacerbation of COPD

A

Haemophilus influenzae

135
Q

Most common cause of gram negative bacterial community acquired pneumonia

A

Klebsiella

136
Q

Most common cause of hospital pneumonias

A

pseudomonas aeruginosa

also commonly causes CAP in patients with cystic fibrosis and immunocompromise

137
Q

Important cause of bacterial secondary infection following viral respiratory illness

A

S. aureus

Another buzzword is - IV users - endocarditis

hospital acquired pneumonia

138
Q

Noncaseating granuloma
bilateral lung and hilar adenopathy
hypercalcemia

What is it and what is treatment?

A

Sarcoidosis

-african american female
stellate inclusions
-bilateral lung and hilar adenopathy (90%)
-mimics Sjorgren syndrome 
-can effect many organs 
-elevated ACE 

Treatment is steroids

139
Q

Organic antigens - moldy hay, thermophilic bacteria, bird

  • fever, cough, dyspnea
  • chronic exposure leads to intersitital fibrosis
A

Hypersensitivity pneumonitis

  • presence of non-caseating granulomas
  • eosinophils
140
Q

Honeycomb lung

  • cyclical epithelial activation/injury
  • TGF beta
  • bleomycin, amiodarone

treatment?

A

Idiopathy pulmonary fibrosis aka usual intersitial pnuemonia

mortality is within 5-6 years 68%.
Only treatment is lung transplant

141
Q

Caplan syndrome

Black lung

A

Breathing in carbon dust

Coal worker’s penumoconiosis

142
Q
  • egg shell calcifications
  • african american
  • fibrotic nodules in upper lobes
  • increase risk of TB
  • bifringent particles
A

-most common occupational lung disease in the world

Silicosis

143
Q

construction worker, plumber, shipyard worker

  • pleura plaques
  • lower lobe
A

Lung cancer

Asbestosis

-smoking severely increase the chances of cancer

144
Q

lecthicin/sphingomyelin <2

  • tachypnea with use of accessory muscles
  • hypoxemia with cyanosis
  • ground glass of lung
  • PDA, necrotizing entercolitis
A

Surfactant production begins at 28 weeks but adequate levels not reached until 34

Neonatal respiratory distress syndrome

145
Q

hypertrophy of bronchial mucinous glands (Reid index >50%)

  • cyanosis
  • increased risk of infection and cor pulmonale
A

chronic bronchitis

at least 3 months of productive cough lasting 2 years

146
Q

Chronic bronchitis = blue bloater?

A

Unlike emphysema, the pulmonary capillary bed is undamaged. Instead, the body responds to the increased obstruction by decreasing ventilation and increasing cardiac output. There is a dreadful ventilation to perfusion mismatch leading to hypoxemia and polycythemia. In addition, they also have increased carbon dioxide retention (hypercapnia). Because of increasing obstruction, their residual lung volume gradually increases (the “bloating” part). They are hypoxemic/cyanotic because they actually have worse hypoxemia than pink puffers and this manifests as bluish lips and faces–the “blue” part.

147
Q
Bronchioles, bronchi 
-necrotizing inflammation
-Cystic fibrosis
Kartegner syndrome 
-Allergic broncho pulmonary aspergillosis (asthma or cystic fibrosis)  
-foul smelling sputum
A

Bronchiectasis

-the air whirls around, because the dilation the air can’t accelerate out

148
Q

What are the 4 stages and times of lung development

and buzzwords of each

A

Embryonic - weeks 4-7: tracheoesophageal fistula, up to tertiary bronchi

Pseudoglandular - weeks 5-16: terminal bronchiole

Canalicular (weeks 16-25): respiratory bronchiole, respiration capable at 25 weeks

Saccular (week 26-birth) - alveoli develop, pneumocytes form, surfactant is getting produced

Alveolar (week 36 - 8 yrs) = surfactant production peaks,

149
Q

Good pasture syndrome

A

rare autoimmune disease of the kidney and lung

  • clinically begins with hemoptysis, rapidly progressive glomerulonephritis
  • focal necrosis of alveolar walls with associated intra-alveolar hemorrhage
  • linear deposits of immunoglobulins and complement along the glomerular basement membrane and alveolar septal walls

Treatment: plasmapheresis, immunosuppressive therapy

150
Q

Idiopathic pulmonary hemosiderosis

A

rare disorder with intermittent diffuse alveolar hemorrhage

  • usually young children
  • productive cough, hemoptysis and anemia
  • favorable response to long term immunosuppression with prednisone
151
Q

Granulomatosis
with polyangiitis
(Wegener)

A

Granulomatosis
with polyangiitis
(Wegener)

autoimmune disease characterized by triad of

  • bilateral nodular and cavitary lung infiltrates, serology for positive c-ANCA is required
  • rapidly fatal
  1. Upper respiratory tract: perforation of nasal septum, chronic sinusitis, otitis media, mastoiditis.
  2. Lower respiratory tract: *hemoptysis, cough, dyspnea.
  3. Renal: *hematuria, red cell casts.

Tr i a d :
ƒ1. Focal necrotizing vasculitis
ƒ2. Necrotizing granulomas in the lung and upper airway ƒ
3. Necrotizing glomerulonephritis

PR3-ANCA/c-ANCA (anti-proteinase 3).
CXR: large cavitary nodular densities.
Treat with cyclophosphamide, corticosteroids

152
Q
central mass 
smoker
hypercalcemia 
keratization pearls 
cavitate
A

Squamous cell carcinoma

the one type of cancer that can cavitate.
Horner’s
Pancoast

153
Q

EGFR

also implicated in which lung cancer?

A

Adenocarcinoma

overexpressed in about 80% of non-small cell cancers

if this intercellular tyrosine kinase domain is mutated, sending signals for the nucleus to divide.

So give tyrosine kinase inhibitors. And stop the division of lung cancer, it has revolutionized the treatment of lung cancer.

154
Q

well differentiated neuroendocrine cells (nest)

  • chromogranin positive
  • central polyp like mass in the bronchus (but can be peripheral)
A

Carcinoid

-low grade malignancy, can cause carcinoid syndrome

155
Q

Buzzwords
lower lobe > apex
-focal, caseating necrosis undergoes fibrosis, calcification
-Later: caseating granulomas
-systemic spread is to any tissue - meninges, cervical lymph nodes, kidneys, lumbar vertebrae

A

tuberculosis

156
Q

what are ppd size cutoffs

A
>=5mm
–  HIV
-former case of TB
-immunosuppressed patient
-close contact has TB

> =10 mm
–  foreign immigrants from country with high incidence
-people from high risk settings (jails, hospitals)
-children < 4 years

15mm
–  healthy people >4 years with low likelihood of TB

157
Q

How to clinically diagnose obstructive and central sleep apnea

What is the definition of apnea
What is hypopnea

What is apnea-hypopnea index
and the numbers associated with mild, moderate, severe

A

Obstructive apnea has the presence of respiratory effort • Central apnea has the absence of respiratory effort

• Apnea= Cessation of flow for ≥10 seconds

• Hypopnea= Decrease in flow by ≥30% for ≥10 seconds associated with
oxygen desaturation or elecrophysiologic arousal

• Apnea-hypopnea index= number of apneas and hypopneas per hour

  • Mild= 5- <15 events per hour
  • Moderate= 15- <30 events per hour
  • Severe= ≥30 events per hour
158
Q

Symptoms of pulmonary embolism along with what three signs rules in the rare fat embolism from long bone fracture

A
  1. Cerebral signs
  2. Petechial rash
  3. Retinal fat globules
159
Q

What is the main agent of injury to the alveolar capillary endothelium in ARDS
What things can cause ARDS
What is the indicative PaO2-FiO2 value?

A

Neutrophil - release chemokines, oxygen radicals, prostaglandins and proteases that damage alveolar epithelium

shock, severe trauma, sepsis, acute pancreatitis, massive transfusion and gastric aspiration are all causes

PaO2-FiO2 <200 mm Hg

160
Q

acute rhinitis

A

Self-limited inflammation
• Most commonly secondary to viral infection
• Release of inflammatory cytokines and chemokines including interleukin (IL)-6, tumor necrosis factor-α, and interferon-γ results in :
– Tissue edema – Mucus production – Vascular dilation
• Nasal irritants, including perfumes, smoke, and cleaning products, can cause a similar constellation of symptoms

161
Q

allergic rhinitis

A

• Initial allergen exposure provokes early and late phase
immune responses
• Early phase:
– Allergen recognition by IgE** subunits on mast calls and basophils
– Degranulation of mast cells and basophils
– Release of pre-formed mediators
– Histamine**
– Tissue edema
– Mucus secretion

• Late phase (4-8 hrs after exposure):
– Infiltration of leukocytes, eosinophils, basophils, CD4 lymphocytes
and monocytes
– Second wave of inflammatory mediators

next time
• Priming of immune response
• Repeated allergen exposure results in amplification of
mucosal hyper-responsiveness
• Due to neural hyper-responsiveness and amplification of mast
cells and basophils

162
Q

Acute rhinosinusitis

A

Acute Rhinosinusitis
• Typically viral etiology
• Between 0.5% - 2% of viral rhinosinusitis episodes will progress to acute bacterial rhinosinusitis
– Viral-mediated mucosal inflammation and edema causes ciliary dysfunction and obstruction of sinus ostia
– Mucus stasis

163
Q

chronic rhinosinusitis

A

Chronic Rhinosinusitis
• Persistent mucosal inflammation
• Numerous theories have been proposed: • Systemic immune dysfunction, Staphylococcal super antigens, Bacterial biofilms, Aberrant immune response to fungus, Dysbiosis (imbalance of the resident microbial population)

• Can have nasal polyposis (eosinophilic infiltrate with TH2
helper cells)
• May have sensitiivy to Aspirin and NSAIDs
• Samter’s triad (Asthma, Chronic rhinosinusitis with polyposis, aspirin/NSAID sensitivity)
• Abnormalities in arachidonic acid metabolism
– Increased inflammatory products of 5-lipoxygenase pathway
– Tx- Steroids

164
Q

Acute or Exudative phase
Organizing phase

Repair and resolution (no time)
or
Chronic fibrosis

Fatal casees

A

Exudative phase (1st week)

– Type I pneumocyte necrosis and sloughing of basement membrane
– Congestion of alveolar capillaries, interstitial and alveolar edema, and intra-alveolar hemorrhage.
– Hyaline membranes in the region of the alveolar ducts.
– Interstitial inflammation –lymphocytes, macrophages
and plasma cells
• End of first week:
– Hyperplasia of the Type 2 pneumocytes that continues throughout the organizing phase

Organizing phase (begins after 1 week)
• Begins towards the end of the first week
• The exudate within the interstitium and alveolar spaces begins to organize • Extensive proliferation of type 2 pneumocytes and fibroblasts along the basement membrane of damaged alveolar septa
• Fibroblasts and myofibroblasts proliferate within the interstitium and migrate through breaks in
the alveolar basement membrane into the intra- alveolar fibrinous exudate.
• Vascular thromboemboli are common • Squamous metaplasia can be seen
-hyaline membranes getting resorbed

Repair and resolution
• Regeneration of type 2 pneumocytes regenerate into type I pneumocytes
– allowing restoration of normal alveolar gas exchange.
• Exudate transforms into granulation tissue that may undergo resorption as the lung returns to normal

Fibrotic chronic phase
• Patients who survive 3-4 weeks on a ventilator
• Extensive remodeling by dense fibrous tissue
• Alveolar spaces and bronchioles may
haphazardly enlarge and are surrounded by dense fibrosis
• Progressive increase in intra-alveolar fibrosis and collagen

In fatal cases, fibrosis progresses for
several weeks with extensive
reconstruction of lung parenchyma and
finally honey comb lung.

165
Q

The steps of ARDS

A
  1. Insult, i.e. sepsis, trauma, severe pancreatitis - sloughing off bronchial epithelium - necrosis of type 1 pneumocytes
  2. Inflammatory cells & cytokines
  3. Capillary endothelial & alveolar epithelial injury
  4. Increased permeability > protein-rich interstitial and alveolar edema
  5. Decreased Surfactant production & function > atelectasis

Buzzwords: acute inflammation, edema, hyaline membranes, • Bilateral pulmonary infiltrates on chest
radiograph

166
Q

What is the number 1 cause of venous hypertension?

A

Valve incompetence due to DVT

Transmits high pressures to the superficial capillary veins.

167
Q

Superficial thrombophlebitis - thombosis plus pain, erythema, induration, tenderness, fever

A

Workup is indicated only if the Great and small saphenous veins are involved.

The reason is not the thrombophlebitis itself but rather the high association with concurrent DVT- especially if ST is near junctions of the deep venous system

There is also risk that these will progress to the deep system.

168
Q

Number 1 cause of spontaneous upper extremity DVT

A

Paget-Schroetter Syndrome: compression of subclavian vein as it is going under the clavicle, mechanical event.

169
Q

Prothrombin G20210A mutation

A

second most common inherited thrombophilic disorder

Mutation is in the 3’ UTR of the prothrombin gene leading to an increase in prothrombin

It is autosomal dominant like protein C deficiency

170
Q

What is the most common thrombophilic inherited disorder

A

Factor V Leiden Mutation

-factor 5 is resistant to activated protein C.

Autosomal dominant,

inherited risk factors for DVT all are along with antithrombin 3 deficiency and protein S deficiency.

171
Q

BMPR2

A

the gene when inactivated is implicated in primary pulmonary hypertension.

bmPR (pulmonary resistance) > leading to proliferation of vascular smooth muscle

Classically seen in young adult females.

Death is within 2 to 5 years and the only treatment is lung transplant.

172
Q

Recurrent emboli and pulmonary hypertension

A

It is a cause of secondary pulmonary hypertension.

Recurrent emboli lead to reduction in the cross sectional area of pulmonary vascular bed.

173
Q

What are the values for defining pulmonary hypertension

A

Right heart catheterization at rest

  1. Mean PA pressure >= to 25 mm Hg
  2. Pulmonary capillary wedge pressure =< 15 mm Hg
174
Q

D-dimer test

A

D-dimer tests are used to help rule out the presence of an inappropriate blood clot (thrombus). Some of the conditions that the D-dimer test is used to help rule out include: Deep vein thrombosis (DVT) Pulmonary embolism (PE)

Its a highly sensitive test so a negative test means you have high certainty no PE. Together with clinical stability and a low Wells Score, they will likely have excellent outcome.

175
Q

What are the classic symptoms of pulmonary embolus

6

A
  1. dyspnea
  2. pleuritic chest pain
  3. Hemoptysis
  4. Hypoxemia
  5. Increase A-a gradient
  6. Respiratory alkalosis (from hyperventilation)
176
Q

What stages of non small cell carcinoma are surgically resectable

A

Stages 1 - 3a

Beyond needs radiation

177
Q

Bronchopulmonary sequestration

A
  • nonfunctioning mass of lung tissue
  • NO communication w/ tracheobronchial tree
  • blood supply from systemic circulation
  • intralobar (75-90%)
  • left lower lobe
  • recurrent infection

Extralobar - outside lung, has its own visceral pleura, congenital anomalies

178
Q

bronchogenic cysts

A

mediastinum location

thin walled structures that can occur anywhere along embryonic airway - lined with respiratory mucosa

Presentation is

  • recurrent infection
  • compression, cough, dyspnea, wheezing
179
Q

Causes of hypoplastic lung

2

A
  1. Congenital diaphragmatic hernia
    - tachypnea, dyspnea, cyanosis, absent bowel sounds, scaphoid abdomen
  2. Potter syndrome - bilateral renal agenesis or bilateral multicystic dysplastic kidney, reduced fetal urine excretion, (lack of) oligohydramnios causing compression
180
Q

primary spontaneous pneumothorax

A
 Tall
 Thin
 Marfan syndrome
 Smoking tobacco
 Smoking marijuana
• Catamenial – associated with menstrual cycle.
  • Alveoli rupture at the point where the alveolar base meets the bronchovascular sheath.
  • Air dissects along the vascular sheaths towards the mediastinum and into the hilum and mediastinal soft tissues.
  • The mediastinal parietal pleura ruptures, and a pneumothorax develops.
181
Q

What are the physical findings of pleural effusion

A

• Physical findings do not usually manifest until pleural
effusions exceed 300 mL
• Dullness on chest percussion
• Decreased tactile fremitus
• Egophony
• Decreased breath sound in chest auscultation
• Decreased lung excursion

182
Q

Lung abscess

A
  • aspiration of infective material:
  • abscess is a form of liquefactive necorsis resulting from neutrophilic enzymatic degradation.
  • a large hole in the lung with accumulated debris along one edge of the cavity.
  • vulnerable patients include those with diminished cough reflex, consciousness, seizure disorders, elderly, alcoholics
  • anaerobic bacteria are commonly cultured (fusobacterium, bacteroides)
  • chest xray reveals a cavitary lesion with an air-fluid level

•  Infection causes necrotic lung tissue to cavitate
•  Bacteria more associated with cavitation/abscess
1–  Actinomyces (Gm+ anaerobe) – alcoholic, poor dental hygiene, post
dental procedure, more in immune competent
2–  Nocardia (Gm+ anaerobe) – (weakly acid fast), seen in soil, immune
compromised

•  Not all cavitations are infectious!
–  Wegener’s, Trauma, cysts, cancer also, among other causes, create pulmonary cavity

•  Pathophysiology
–  Small zones of necrosis in consolidated regions of pneumonia form single or multiple abscesses that erode into bronchi, ultimately resulting in fibrosis

183
Q

13 yr old boy has history of chronic sinusitis, bronchitis, and recurrent otitis media. A sweat chloride test is performed twice and results both times are nromal. Microscopic anlaysis of the boy’s semen reveals immotile speratozoa. A defect in what protein is responsilbe

A

Defects in the cytoskeletal structure of cilia, particularly dynein.

Sweat test: A sweat test measures the amount of salt chemicals (sodium and chloride) in sweat. It is done to help diagnose cystic fibrosis. Normally, sweat on the skin surface contains very little sodium and chloride. People with cystic fibrosis have 2 to 5 times the normal amount of sodium and chloride in their sweat. The defective chloride channel prevents reabsorption of ions.

During the sweat test camera.gif, medicine that causes a person to sweat is applied to the skin (usually on the arm or thigh). The sweat is then collected on a paper or a gauze pad, and the amount of salt chemicals in the paper or gauze is measured in a lab. Generally, chloride (sweat chloride) is measured.

184
Q

What artery is responsible for epistaxis

A

The sphenopalatine artery is the major blood supply to the nasal mucosa
= it is a terminal branch of the maxillary artery

185
Q

How do these parameters change from living at high altitude

Arterial PO2
Arterial O2 saturation
Hematocrit

A

Arterial PO2: decrease
Arterial O2 saturation - decrease
Hematocrit - increase

  • arterial PO2 and arterial O2 sat both reamin low because these parameters are both virtually independent of blood hb concentration. The oxygen saturation curve yo
186
Q

Transesophageol fistula

A

-abnormal communication between the trachea and esophagus caused by a malformation of the tracheoesophageal septum

It is generally associated with esophageal atresia and polyhydramnios because of fetus has difficulty swallowing.

Results in immediate vomiting after feeding, copious, frothy secretions (air can enter the stomach) and gagging

  • a nasogastric tube is placed and resistance if felt after several centimeters.
187
Q

Hilar lymphadenopathy, multiple tiny oval forms within macrophages. Pennsylvania

A

Histoplasmosis

188
Q

What is in a PPD test?

A

purified protein derivative - tuberculin (outer surface protein) and mycolic acids of the organism.

189
Q

Nasopharyngeal carcinoma

A
buzzwords"
Southern China adult
Africa childhood
EBV
enlarged cervical lymph nodes 
-pleomorphic keratin cells over a background of lymphocytes
190
Q

A 25-year-old woman with a history of asthma
is brought to the emergency department by
emergency medical services (EMS) after ingesting
a full bottle of theophylline in a suicide
attempt. At presentation, she is having a tonicclonic
seizure. Her blood pressure is 80/40
mmHg, respiratory rate is 30 breaths/minute,
and her heart rate is 160/minute. The EMS
personnel report that she has been seizing for
at least 15 minutes. What is the mechanism of
action of the most appropriate drug to counteract
her intoxication?
(A) Decreasing intracellular cAMP
(B) Increasing intracellular cAMP through
b2-adrenergic receptors
(C) Increasing intracellular cAMP through
nonselective adrenergic receptors
(D) Inhibiting the Na+-K+-2Cl- co-transporter
(E) Inhibiting the Na+-K+-ATPase pump

A

A

191
Q

A 35-year-old HIV-positive woman with a
CD4+ count of 175/bL (normal = 500-1500/
bL) presents to the clinic with a two-week history
of fever, nonproductive cough, and progressive
dyspnea. She has a history of sulfa allergy. Physical examination reveals diffuse
crackles and rhonchi. X-ray of the chest shows diffuse, bilateral interstitial infiltrates. Laboratory
studies are remarkable only for an elevated
lactate dehydrogenase level. Which of
the following is the best choice for prophylaxis
against this infection in a patient with a sulfa
allergy?
(A) Aerosolized pentamidine
(B) Ciprofloxacin
(C) Fluconazole
(D) Terbinafine
(E) Trimethoprim-sulfamethoxazole

A

This vignette suggests
a patient with Pneumocystis jiroveci pneumonia based on the HIV status, physical
examination and x-ray findings, and the
elevated lactate dehydrogenase level. Prophylactic therapy for P jiroveci pneumonia is indicated for an HIV-positive patient with a CD4+ T-lymphocyte count <200/μL. The standard prophylactic therapy for P jiroveci pneumonia is trimethoprim-sulfamethoxazole (TMP-SMX). This combination, however, is contraindicated for patients with a sulfa allergy,
because sulfamethoxazole is a sulfa drug.
In these cases, the best alternative treatment is
aerosolized pentamidine

192
Q

A previously healthy 41-year-old man misses
several days of work as a result of a viral illness
with symptoms including fever, headache, and
fatigue. He also experiences a nonproductive
cough and a sore throat. By the third day, his
symptoms begin to subside and he is able to
return to work. The next week, however, he
experiences a rapid relapse. His cough returns,
but now it is a productive cough with mucopurulent
sputum. He also begins to experience
pleuritic chest pain. On visiting a physician,
x-ray of the chest is ordered and is shown in
the image. Which of the following describes
the structure of the viral genome that most
likely caused his initial illness?

Image shows a right lower lobe pneumonia

(A) Enveloped, double-stranded DNA
(B) Enveloped, nonsegmented, single-stranded
RNA
(C) Enveloped, segmented, single-stranded
RNA
(D) Non-enveloped, nonsegmented, doublestranded
RNA
(E) Non-enveloped, nonsegmented, singlestranded
RNA

A

From the history,
it appears that this man initially experienced
nonspecific viral symptoms, but there is not
enough information to determine which virus
he has. What is clear, however, is that his initial
symptoms are distinct from what he experiences
on relapse. The radiograph shows that he has lobar pneumonia (lower right lobe),
which can be caused by any number of bacterial
species. The question that must be asked,therefore, is, “Which viral illness predisposes
to subsequent bacterial pneumonia in an otherwise
healthy individual?” The classic answer
is influenza. The influenza virus is an
enveloped, single-stranded RNA virus with a
segmented genome that permits reassortment
of the genes encoding the hemagglutinin and
neuraminidase proteins, resulting in the phenomenon
of antigenic shift. Complications of
influenza include both viral pneumonia (due
to a spreading of the illness into the lower respiratory
tract) and bacterial pneumonia. The
latter is thought to be due largely to the fact
that influenza damages the epithelium of the
upper respiratory tract, compromising its ability
to keep the lower respiratory tract sterile.
Streptococcus pneumoniae, Staphylococcus
aureus, and Haemophilus influenzae are the
organisms most commonly seen in bacterial
pneumonia secondary to influenza. This chest
x-ray shows a consolidation in the right lower
lobe along with a para-pneumonic effusion,
highly suspicious for bacterial pneumonia.

193
Q

A 25-year-old medical student presents to the
clinic with a nonproductive cough, low-grade
fever, and malaise of three weeks’ duration. A
few friends in his study group have been feeling
the same way. Sputum cultures are negative.
The patient denies exposure to farm animals,
travel, or HIV. The physician treats for
an atypical pneumonia. Which of the following
methods could help identify the organism
responsible for this most likely causative pathogen?
(A) Acid-fast stain
(B) Cold agglutinin testing
(C) Gram stain
(D) India ink stain
(E) Serum polymerase chain reaction

A

In patients who
present with insidious onset of dry cough, low-grade fever, headache, myalgias, nausea, or emesis, an atypical pneumonia should be considered. Atypical pneumonias are mostly
caused by Mycoplasma or viruses. Mycoplasma
cannot be cultured and is detected by the cold
agglutinin test, which measures the agglutination
of immunoglobulins when they are cooled. X-ray of the chest is often more impressive than physical examination findings, and is characterized by a patchy interstitial pattern.
Treatment consists of antibiotic therapy with a macrolide, usually azithromycin, for five days

194
Q

A 2-year-old boy presents to the emergency department
because of sore throat, fever, hoarseness,
and stridor. The physician suspects a
diagnosis of croup, but wishes to exclude epiglottitis.
Compared with croup, which of the
following is characteristic of epiglottitis?
(A) Epiglottitis is associated with inflammation
of the larynx and subglottic trachea
(B) Epiglottitis is associated with rhinorrhea
and conjunctivitis
(C) Epiglottitis often leads to respiratory distress
(D) Symptom onset is gradual in epiglottitis
(E) The barking cough of epiglottitis becomes
inspiratory stridor
(F) Throat swab in epiglottitis would reveal
parainfluenza virus

A

.Epiglottitis is a medical
emergency, and 90% of patients require surgery to reestablish an airway. At presentation patients with epiglottitis can have little
or no respiratory compromise, but this can
progress to life-threatening respiratory distress
within a matter of hours.

Epiglottitis on x-ray
film of the neck reveals a “thumbs up” sign (ie,
“thumbprint” on radiograph), which correlates with an inflamed epiglottis. Inflammation of
the larynx and subglottic trachea is not associated
with epiglottitis.

Patients with epiglottitis
do not have the symptoms or physical findings
of conjunctivitis or rhinorrhea. These
findings are more typical of croup. Epiglottitis
has additional symptoms of drooling and labored
breathing.

In general, the onset of
symptoms is abrupt with epiglottitis and gradual
with croup.

A typical barking cough
is seen with croup, which may eventually lead
to inspiratory stridor. Epiglottitis typically presents with stridor and hoarseness.
Most of the time, a throat
swab in epiglottitis will reveal Haemophilusinfluenzae, not parainfluenza. Parainfluenza is more often seen in croup.

195
Q

A 25-year-old man presents with new-onset
hemoptysis for the past 12 hours. The patient,
who recently immigrated to the United States
from Vietnam, has had fever and night sweats
on a daily basis for the past four years. He has
no other complaints and no past medical history.
The patient is diagnosed with active tuberculosis
and sent home on an antimycobacterial
regimen. One month later, the patient
returns with new complaints of joint pain,
photosensitivity, and a facial rash. Liver function
tests are found to be elevated. Which of
the following is the mechanism of action of the
drug with the adverse effects described above?
(A) Disrupts the cell membrane’s osmotic
properties
(B) Inhibits arabinosyl transferases
(C) Inhibits DNA-dependent RNA polymerase
encoded by the rpo gene
(D) Inhibits folic acid synthesis
(E) Inhibits synthesis of mycolic acids

A

.Of the antimycobacterial
drugs, only isoniazid (INH) produces the
lupus-like syndrome described above. INH
decreases synthesis of mycolic acids. Furthermore,
hepatotoxicity is common to many antituberculosis
drugs (rifampin, pyrazinamide,
and INH).

196
Q

A 57-year-old man presents to his primary care
physician for a routine wellness check. He
denies any complaints. Social history is significant
for a 50-pack-year smoking history. On
physical examination, his vital signs are within
normal limits except for his blood pressure,
which is 170/95 mm Hg. On his previous visit,
his blood pressure was 155/90 mm Hg. Which
of the following antihypertensive agents is relatively contraindicated in this patient?
(A) Acebutolol
(B) Atenolol
(C) Esmolol
(D) Metoprolol
(E) Nadolol

A

..Nonselective bblockers
are contraindicated in patients with
lung disease because they can cause bronchoconstriction
by blocking b2-receptors responsible
for relaxation of bronchial smooth muscle.
Nadolol is a nonselective b-blocker and should
not be used in a patient with lung disease.
Other nonselective b-blockers include propranolol, timolol, and pindolol. Acebutolol, atenolol, esmolol, metoprolol, and betaxolol are cardioselective b1-blockers that should be favored in patients with lung/airway disease. Although the stem does not specifically state that
this patient has lung disease, smoking causes
airway hyperreactivity and bronchoconstriction.
Adding a nonselective b-blocker could exaggerate
these adverse effects of smoking.

197
Q

A 50-year-old woman complains of dark colored
urine and says she has not been feeling well for the past two-three weeks; she has generalized malaise and a nagging cough that
occasionally is productive of blood-tinged
sputum. However, she noticed changes in
her urine for the first time today. Physical examination
reveals an ill-appearing middle-aged
woman with a blood pressure of 180/110 mm
Hg. Diminished air entry in the lungs bilaterally,
and an ulcerated lesion of the mucosa of
the right naris, are noted. There is no history
of asthma or allergies. Urinalysis is grossly positive vfor blood, and serum chemistry panel reveals a creatinine level of 1.7 mg/dL. What additional finding would confirm the most likely
diagnosis?
(A) Eosinophilia on WBC differential
(B) IgA deposition in glomerular mesangium
(C) Linear IgG deposition in the kidney
(D) Positive for cytoplasmic anti-neutrophilic
cytoplasmic antibodies
(E) Positive for hepatitis B

A
This patient’s constellation
of symptoms is most consistent with Wegener granulomatosis, with the triad of 
1. focal necrotizing vasculitis, 
2. necrotizing granulomas
of the upper and/or lower airways, 
3. necrotizing glomerulonephritis. 

Most patients have positive titers for anti-neutrophil cytoplasmic antibodies with a cytosolic staining pattern
(c-ANCA). The disease is caused by systemic
granulomatous inflammation, particularly of small- and medium-sized arteries such as those
supplying the kidneys and lungs. If not treated
with immunomodulating drugs, focal glomerulonephritis
can progress to a crescentic form, with ensuing renal failure.

-dark colored
urine
- productive of blood-tinged
sputum.
-creatinine level of 1.7 mg/dL
- ulcerated lesion of the mucosa of the right naris
198
Q

A 35-year-old African-American woman presents
to the clinic complaining of fatigue, dry cough, and dyspnea. X-ray of the chest is shown in the image. Lung parenchymal biopsy
reveals a noncaseating granuloma. Which of
the following cutaneous manifestations is associated
with this condition?
(A) Erythema infectiosum
(B) Erythema migrans
(C) Erythema multiforme
(D) Erythema nodosum
(E) Erythema toxicum

A

.The patient has signs
and symptoms of sarcoidosis, with classic race, pathology (noncaseating granuloma), and x-ray of the chest revealing prominent bilateral hilar
lymphadenopathy, which is present in >90% of patients with sarcoidosis. Erythema
nodosum, an inflammatory panniculitis, is the most common cutaneous manifestation of sarcoidosis,
and frequently presents as bilateral tender red bumps on the shins. Additional
features of sarcoidosis include hypercalcemia
due to increased activation of vitamin D by activated
macrophages.

199
Q

A 5-year-old girl visiting from Mexico is brought to the emergency department by her aunt because of a sore throat and general malaise
for the past three days. Physical examination
reveals temperature of 38°C (100.4°F) and a grayish-white membrane on the pharynx
that bleeds on attempted dislodgement. Which
of the following is the most appropriate culture
media for diagnosing this patient’s infection?
(A) Bordet-Gengou agar
(B) Chocolate agar with factors V and X
(C) Sabouraud agar
(D) Tellurite agar
(E) Thayer-Martin agar

A

.This patient likely
has diphtheria, an infection caused by the
gram-positive rod Corynebacterium diphtheriae.
Diphtheria classically presents with a
grayish-white pseudomembrane on the pharynx or tonsils; this pseudomembrane should
not be disrupted in order to avoid increased
absorption of the lethal exotoxin. Fever is usually
mild or absent. It is seen very rarely in vaccinated
populations but is endemic to certain parts of the world. Culture of C diphtheriae
requires tellurite agar (Loeffler medium) to
prevent growth of normal upper respiratory
tract flora. Colonies will become gray to black
within 24 hours.

Bordet-Gengou agar is
used to culture Bordetella pertussis. Pertussis
presents with paroxysmal coughing spells and
whooping sounds on inspiration.

Chocolate agar is used
to grow Haemophilus influenzae. Encapsulated
strains of H influenzae cause invasive diseases
such as septicemia, meningitis, cellulitis, septic
arthritis, epiglottitis, and pneumonia. Nonencapsulated
strains are likely to cause otitis
media, conjunctivitis, bronchitis, and sinusitis.

Sabouraud agar is used
to grow fungi.

Thayer-Martin agar is
a chocolate agar plate, which has VCN antibiotics
(vancomycin, colistin, and nystatin) that suppress the growth of endogenous flora while supporting Neisseria gonorrhoeae growth. This
patient does not have symptoms of gonorrhea

200
Q

A 57-year-old man presents to the emergency
department (ED) with fever, night sweats, and
a productive cough with occasional hemoptysis.
He is started empirically on several medications
for his underlying disease. At follow-up
several months later, he reports difficulty reading
the paper in the morning and has been
found to wear unusual color combinations at
work. Which of the following is the most likely
cause of this patient’s new symptoms?
(A) Ethambutol toxicity
(B) Isoniazid toxicity
(C) Pyrazinamide toxicity
(D) Rifampin toxicity
(E) Tuberculous eye infection m

A

Ethambutol is active
against Mycobacterium tuberculosis, and it
is among the first-line agents used to treat tuberculosis
(TB) infection (others are isoniazid,
rifampin, and pyrazinamide). Ethambutol’s
mechanism of action appears to be the inhibition
of polymerization of cell-wall precursors.
Although the drug generally is well tolerated,
its most common adverse effects involve ocular
toxicity such as loss of visual acuity and redgreen
color blindness, which usually appears
several months after the initiation of treatment.
Ethambutol usually is used in an anti-TB
regimen with rifampin for patients who either
cannot tolerate isoniazid or are infected with
isoniazid-resistant M tuberculosis. For children,
most literature supports a regimen of six
months with isoniazid and rifampin, with ad

201
Q

A 36-year-old woman with a history of leukemia
receives a bone marrow transplant. Two
and a half weeks later, she experiences fever,
cough, and dyspnea. Bronchoalveolar lavage
reveals large cells with prominent intranuclear
inclusions,

A

.Cytomegalovirus
(CMV) infection is a common complication
in immunocompromised patients following
bone-marrow transplantation. Histopathology
shows large cells with intranuclear inclusions
(so-called “owl’s eyes”) typical of CMV infection.

202
Q

A 12-year-old boy is found unconscious in his
bedroom by his parents and is taken to the
emergency department. On arrival the patient’s
skin is pale and lacks turgor, and there is
a sweet scent on his breath. His parents report
constant urination and weight loss in the two
weeks prior to presentation. Laboratory tests
show a glucose level of 610 mg/dL, sodium
of 130 mEq/L, bicarbonate of 9 mEq/L, and
chloride of 95 mEq/L. Which of the following
would most likely be associated with this patient’s
condition at presentation?
(A) Calcium oxalate crystals in the urine
(B) Decreased anion gap
(C) Decreased blood partial pressure of carbon
dioxide
(D) Elevated blood partial pressure of carbon
dioxide
(E) Hypokalemia

A

.This patient most
likely has type 1 DM and a resulting ketoacidosis.
He has a metabolic acidosis with a large anion gap (>10 mEq) as calculated by the following formula: Anion gap = Na+ - [HCO3
- + Cl-]. This leads to respiratory compensation
by deep respiration (Kussmaul’s respiration),
resulting in a decrease in blood partial pressure
of carbon dioxide. The large anion gap is due to the overproduction of ketones in the absence of insulin production.

203
Q

A 46-year-old woman presents to the emergency
department because of a one-week history
of worsening nausea and lethargy. While she is waiting to see the doctor she experiences a seizure. Her past medical history is significant for tuberculosis. Laboratory values show:
Serum Na+: 109 mEq/L
Serum osmolality: 255 mOsm/kg
Urine osmolality: 850 mOsm/kg
Hematocrit: 27%
Which of the following drugs is also known to
cause the underlying disorder with which this
patient presents?
(A) Cyclophosphamide
(B) Demeclocycline
(C) Hydrochlorothiazide
(D) Indomethacin
(E) Lithium

A

.The patient is suffering
from the syndrome of inappropriate ADH
secretion (SIADH), a condition in which excessive ADH is secreted independently of serum
osmolality; this can be seen in a variety of pulmonary diseases (including TB) as well as central nervous system (CNS) disorders that
enhance ADH release (eg, stroke, hemorrhage,
infection, and trauma) and certain carcinomas
(most commonly small cell lung carcinoma).
SIADH can also be an adverse effect of some
drugs, notably high-dose intravenous cyclophosphamide.
Other drugs shown to cause SIADH include carbamazepine, vincristine,
vinblastine, cisplatin, amitriptyline, amiodarone,
and monoamine oxidase inhibitors.
Excessive ADH secretion can lead to nausea,
lethargy, seizures, and even coma. The patient’s
laboratory values are typical of someone
with SIADH, showing hyponatremia, serum
hypo-osmolality, urine hyperosmolarity, and
decreased hematocrit secondary to dilution.

Like lithium, demeclocycline
has been associated with nephrogenic diabetes insipidus (DI). Demeclocycline is
not known to cause SIADH, the underlying
disorder that accounts for this patient’s presentation.
In fact, demeclocycline is sometimes
used to treat SIADH.

Hydrochlorothiazide
(HCTZ) is a diuretic that may sometimes result
in hyponatremia. However, this patient is presenting with SIADH, and HCTZ is not a known cause of SIADH. HCTZ would be unlikely to account for the degree of hyponatremia
(as well as the other laboratory abnormalities)
seen in this patient. Like indomethacin, HCTZ can be used to treat nephrogenic DI.

Indomethacin has not
been associated with SIADH. It is, however,
sometimes used to treat nephrogenic DI. Patients
with nephrogenic DI typically present
with serum hyperosmolality, hypernatremia,
and urine hypoosmolality.
Answer E is incorrect. Lithium toxicity has
been shown to cause nephrogenic DI rather
than SIADH. In nephrogenic DI the kidneys
are unable to absorb water appropriately in response
to ADH. Patients present with production
of large quantities of dilute urine, serum
hyperosmolality, and hypernatremia. They
report both polydipsia and polyuria. Medical
treatment of nephrogenic DI may consist of
hydrochlorothiazide or indomethacin.

204
Q

A 17-year-old girl involved in a car accident
presents to the emergency department with
penetrating chest trauma to her left side. She
is having difficulty breathing and has an oxygen
saturation of 86%. After x-ray of the chest
is performed, a chest tube is placed, and her
oxygen saturation improves. Which of the following
is responsible for her difficulty breathing
upon presentation?
(A) Her intrapleural pressure is equal to atmosphere
pressure during inspiration
(B) Her intrapleural pressure is less than atmospheric
pressure during inspiration
(C) Pain from the trauma has made it difficult
to breathe
(D) Pressure within the pericardial space is increased
relative to the pleural space
(E) The elastic force of the chest wall is pulling
it inward

A

.The patient’s penetrating
chest wound opened her intrapleural space to the atmosphere. Therefore, as she attempts to inhale, her thoracic cavity expands but air enters through the wound, equalizing
the pressure; this prevents the normal expansion
of the lungs. If air is not able to escape through the wound duration exhalation, this is
called a tension pneumothorax, in which the
quantity of free air in the thoracic cavity increases
after each breath.

205
Q

A 158.8-kg (350-lb) man with a body mass
index of 40 kg/m² comes to the physician
complaining of frequent fatigue, shortness of
breath, general sleepiness, and an inability to
concentrate. Physical examination shows an
extremely obese, tired-looking man with hypertension
and an elongated uvula. Which of the
following metabolic findings is most likely?
(A) Decreased serum glucose
(B) Increased HDL cholesterol
(C) Increased renal H+ reabsorption
(D) Increased renal HCO3- reabsorption
(E) Increased renal HCO3- secretion

A

This man is likely
suffering from obstructive sleep apnea (OSA)
secondary to extreme obesity (Pickwickian syndrome).
During the night he has intermittent cessation of airflow at the nose and mouth.
During this progressive asphyxia, he has a brief
arousal, restores airway patency, and returns
to sleep. This patient’s obesity and elongated uvula are very good indicators of OSA, as are
his daytime sleepiness, inability to concentrate,
and hypertension. Periodic, recurrent asphyxia
has the effect of causing a respiratory acidosis
that, when present chronically, is compensated
for by renal retention of HCO3
-.

206
Q

A 68-year-old man who smokes and is alcoholic
abruptly develops high fever, shakes, a severe headache, and abdominal and muscle
pain. He initially has a dry, insignificant cough, but over the next few days he develops marked shortness of breath requiring assisted
ventilation. X-ray of the chest demonstrates homogenous
radiographic shadowing involving both the lungs extensively. Culture of bronchoalveolar lavage fluid on buffered charcoal yeast extract demonstrates a coccobacillary pathogen. What is the most likely causative organism?
(A) Legionella pneumophila
(B) Listeria monocytogenes
(C) Spirillum minus
(D) Staphylococcus aureus
(E) Streptococcus pneumoniae

A

.Legionella pneumophila

The patient has a severe,
potentially fatal, pneumonia with prominent
systemic symptoms. Culture on buffered
charcoal yeast extract is the specific clue that
the organism is Legionella pneumophila. The
disease is respiratory legionellosis, also known
as legionnaire’s disease, so named because the
disease was first described when it occurred in
epidemic form after an American Legion convention
at a Philadelphia hotel. Patients tend
to be older and may have risk factors, including
cigarette use, alcoholism, diabetes, chronic
illness, or immunosuppressive therapy.

207
Q

A 61-year-old man is frustrated because he
is no longer able to walk up a flight of stairs
without stopping to catch his breath. He has
also been plagued by a dry cough for the past
six months. He has not visited his primary
care physician because he is not a smoker and
does not believe that he could have a serious
pulmonary condition. He ignores his symptoms
for another eight months, during which
time they continue to worsen. He finally visits
his physician at the urging of his wife. While
shaking hands, his physician notices that the
patient has clubbing of the fingers. A clinical
work-up and medical history fail to find a
cause for this restrictive lung disease. What is
the definitive therapy for this patient’s most
likely condition?
(A) Albuterol
(B) Azathioprine
(C) Cyclosporine
(D) Lung transplantation
(E) Steroids

A

. Interstitial (or idiopathic)
pulmonary fibrosis (IPF) is a chronic,
progressive fibrotic disorder of the lower respiratory
tract that affects older adults. It is
characterized by the abnormal proliferation
of mesenchymal cells, disruption of collagen
structures, and impaired gas exchange. The
exact pathogenesis of IPF is still unknown. If
not treated, IPF often results in death within
five years. Lung transplantation is currently
the only “cure” for this disease. On x-ray of
the chest, IPF usually is seen best in the lower
parts of both lungs as white lines in a netlike
pattern

208
Q

Public health investigators are looking into a
series of illnesses that have occurred in a small
community. Many patients presented with acute-onset hyperpyrexia and a particularly severe
pneumonia. Gram staining of their sputum cultures reveals neutrophils and very few organisms. Which of the following organisms is most likely to have caused this outbreak?
(A) Bordetella pertussis
(B) Haemophilus influenzae type B
(C) Legionella pneumophila
(D) Mycobacterium tuberculosis
(E) Streptococcus pneumoniae

A

Legionella pneumophila
is an aerobic, gram-negative rod that causes Legionnaire’s disease, a condition in
which patients develop acute, severe pneumonia
and a high fever. Other signs and symptoms include hyponatremia (which is unique
to this pneumonia) and CNS changes. Legionnaire’s
disease is one of the most common causes of community-acquired pneumonia but
is identified as the cause in only 3% of cases.
The organism is present only in water sources
(eg, air conditioning systems, whirlpools, mist
sprayers) and causes infection when aerosolized
water droplets are inhaled. Transmission
is not by person-to-person contact. Typically,
more severe illness is seen in patients who are
>50 years of age, those who smoke, and those
whose Gram stain shows neutrophils and very
few organisms, as in this case. Treatment is
with erythromycin, because L pneumophila
produces a b lactamase that renders it resistant
to penicillin derivatives.

209
Q

A 26-year-old recent immigrant from Mexico
presents to the emergency department with a
three-week history of fevers accompanied by
night sweats and chills, weight loss of 2.3 kg
(5 lb), and cough that is often productive of
blood-tinged sputum. Bronchoalveolar lavage
is performed and an acid-fast stain of the sample
reveals the organism shown in the image.
Which of the following should be included in
this patient’s therapy to prevent a common toxicity
of treatment?

A

.The patient is suffering
from TB, with the causative organism (Mycobacterium tuberculosis) seen as red on
acid-fast stain. Symptoms include fever, night
sweats, chills, cough, and weight loss. His treatment
regimen will include isoniazid and other
antimycobacterial agents. Isoniazid inhibits
mycolic acid synthesis in the mycobacterial
cell wall. Because of rapid development of resistance,
isoniazid should never be used alone to treat active TB. Isoniazid depletes pyridoxine (vitamin B6), which is required for the
production of dopamine, epinephrine, norepinephrine,
and monoamine neurotransmitters.
Hence, one of the adverse effects of isoniazid
therapy is peripheral neuropathy, which can
be prevented by co-administration of vitamin
B6.

210
Q

A mother brings her 10-year-old son with fever,
cough, and difficulty breathing to the emergency
department. Approximately two days ago she noted the development of a rash on
his face that spread downward over his body.
Physical examination reveals a toxic-appearing
child with a temperature of 40°C (104°F),
rapid pulse, and rapid respiratory rate. The
physician notes the appearance of a reddishbrown
blotchy rash throughout the child’s
body. In his mouth he has small red spots with
blue-white centers. Chest examination reveals
clear breath sounds with poor inspiratory effort.
CT of the chest shows diffuse interstitial
involvement. Which of the following would
the physician most likely see in this child’s sputum?
(A) Acid-fast bacilli
(B) Cells with nuclei surrounded by halo and
clear cytoplasm
(C) Cowdry-type inclusions in cells
(D) Gram-negative coccobacilli and polymorphonuclear
leukocytes
(E) Gram-positive diplococci and polymorphonuclear
leukocytes
(F) Multinucleated giant cells

A

This child has measles
complicated by pneumonia. Pneumonia complicates approximately 4% of measles cases in the United States and as many as 50% of
cases abroad. Clinically, this child has a high
fever, Koplik’s spots, maculopapular rash, and
CT of the chest showing diffuse interstitial involvement.
Measles-infected respiratory cells
will fuse and form multinucleated giant cells,
which can be detected in sputum samples.
Measles is a member of the Paramyxoviridae
family, a group of negative-sense, singlestranded
RNA viruses. In immunocompromised
hosts, measles pneumonia may evolve to giant cell pneumonia, which is often fatal

211
Q

A 63-year-old smoker visits his primary care physician because of recent weight gain and worsening coughs. On physical examination
the physician notes that the patient’s extremities
are thinner than before, while his waist is increased in girth. The patient also has a pad
of adipose tissue at the base of his neck and purple striae on his abdomen. The physician
decides to run some blood tests and obtain an
x-ray of the chest, which shows a central lesion.
Which of the following is the most likely
diagnosis?
(A) Adenocarcinoma
(B) Bronchial carcinoid
(C) Metastatic disease affecting the lung
(D) Small cell carcinoma
(E) Squamous cell carcinoma embolism

A

.This patient is showing
signs of Cushing syndrome with a buffalo
hump and purple striae. Cushing syndrome
is caused by an excess of cortisol either because
of a pituitary adenoma producing excess
ACTH, an adrenal adenoma producing excess
cortisol, or ectopic ACTH production by
a neoplasm. This man’s smoking history and
lung nodule shown on chest radiography point
to lung cancer. Taken together with ectopic
production of ACTH, this patient has paraneoplastic
syndrome, with ectopic production
of ACTH by the malignant lung mass. Of the
different histological classifications of lung
cancer listed above, small cell carcinoma is
the most likely in the case for several reasons:
Squamous cell and small cell carcinomas are
most closely linked to smoking history (>98%
are associated with smoking) and both present
as central lesions such as that shown on
the x-ray film. Additionally, tumors producing ACTH or ADH are usually small cell carcinomas.

212
Q

A 37-year-old man is brought to the emergency
department after being stabbed superior to his
right nipple with a knife. His blood pressure is
100/60 mm Hg, heart rate is 126/min, respiratory
rate is 26/min, and oxygen saturation is
90% on 100% oxygen facemask. The wound is bubbling, and the skin immediately around
the wound is moving in and out with respirations.
Which of the following will most likely
be found on the right side during x-ray of this
patient’s chest?
(A) Hemothorax
(B) Ninth rib fracture
(C) Pleural effusion
(D) Pneumothorax
(E) Upper lobe consolidation

A

.This question requires
knowledge of both the anatomy and the
physiology of the sucking chest wound, as described
in this patient. A penetrating wound
to the chest can puncture the pleura, making
an opening for air to be sucked into the pleural
space. With inspiration, the diaphragm
descends, lowering the intrapleural pressure.
If there is a communication directly between
the pleural space and the outside world, air is
sucked into this negative-pressure space and
collapses the lung. Pneumothorax is seen on x-ray of the chest as a collapsed lung with a
mediastinum shifted away from the collapsed
lung. With pneumothorax, the patient should
be assessed for signs and symptoms of hemodynamic
compromise. This patient, for example,
is hypotensive, tachycardic, and tachypnic,
and therefore requires urgent management.

213
Q

A 15-year-old boy with a history of severe
asthma presents to the emergency department
in obvious respiratory distress. After multiple
nebulizer treatments and doses of intravenous
corticosteroids, he develops nausea, vomiting,
and weakness. Studies reveal a potassium level
of 2.6 mEq/L and U waves on ECG. Which of
the following medications most likely would
have elicited these symptoms?
(A) Albuterol
(B) Cromolyn
(C) Ipratropium
(D) Theophylline
(E) Zileuton

A

B-agonists such as
albuterol may cause potassium to shift into
cells, resulting in hypokalemia. This may lead
to ECG abnormalities due to destabilization of
cardiac cell membranes, the classic examples
of which are U waves. Short-acting b-agonists
such as albuterol are used in the treatment of
acute asthma exacerbations because of their
relaxing effects on bronchial smooth muscle.
Long-acting β-agonists such as salmeterol are
used for prophylaxis of bronchospasm.

214
Q

A 32-year-old man returns from an in-depth
tour of a sheep and goat farm. Five days later
he develops fever, malaise, a dry cough, and
pressure in his chest. These symptoms resolve
after a few days. He then develops high
fever, severe shortness of breath, chest pain,
cyanosis, and diaphoresis and is rushed to the
emergency department, where work-up reveals
hemorrhagic mediastinitis, bloody pleural effusions,
and mediastinal widening on x-ray of the
chest. Within a few hours the patient develops
septic shock and dies. Which of the following
characterizes the most likely causative organism?
(A) Gram-negative pleomorphic aerobic coccobacilli
(B) Gram-positive rods in chains with a protein
capsule
(C) Gram-negative pleomorphic coccobacilli
requiring cysteine for growth
(D) Gram-positive weakly acid-fast rods forming
long branching filaments
(E) Poorly stained gram-negative rods that
stain best on silver stain and require iron
and cysteine.

A

This describes Bacillus
anthracis, which can cause cutaneous anthrax,
inhalation anthrax, and GI anthrax. This patient had inhalation anthrax (also known as
“wool-sorter’s disease”), which usually has two
phases: the initial phase characterized by malaise,
dry cough, and chest pressure that resolve
in a few days; and the second phase in which
patients suddenly develop acute respiratory distress
and hypoxemia followed by hemorrhagic
mediastinitis and bloody pleural effusions. A
classic radiologic finding is mediastinal widening.
If a patient is not rapidly treated with penicillin,
doxycycline, ciprofloxacin, or levofloxacin, systemic infection can cause septic shock
(due to exotoxins produced by the bacteria)
and death within 24 hours. Spores from sheep
or goat skin are the primary mode of transmission
in this kind of anthrax. Interestingly,
B anthracis is the only medically relevant bacteria
with a protein capsule.

215
Q

A 28-year-old smoker presents to the emergency
department because of sudden onset of
chest pain and dyspnea while at rest. His heart
rate is 115/min, respiratory rate is 24/min, and
blood pressure is 140/80 mm Hg in both arms.
Lung examination shows decreased breath
sounds and decreased fremitus on the right
with hyperresonance to percussion. Which of
the following would most likely be seen on this
patient’s x-ray of the chest?
(A) A widened mediastinum
(B) Barrel chest and flattened diaphragm
(C) Consolidation in the right lower lobe
(D) Contralateral deviation of the trachea
(E) Tracheal deviation to the ipsilateral side;
elevated diaphragm on the right side

A

This patient is most
likely suffering from a spontaneous pneumothorax.
Caused by the rupture of a small apical bleb on the surface of the lung, spontaneous pneumothoraces typically present in tall young
men. The patient usually has sudden pain and dyspnea. Examination will show decreased breath sounds and hyperresonance on the affected side. X-ray of the chest shows overexpansion
of the rib cage and an elevated hemidiaphragm
on the affected side. This paradoxical abdominal motion occurs because of the negative intrathoracic pressure that causes the fatigued diaphragm to be pulled into the thorax on the right side. Spontaneous pneumothorax is treated by inserting a chest tube to remove the air from the pleural space.
It is possible that this
man could have emphysema due to an a1-antitrypsin deficiency, but in that case it would not present this acutely.

216
Q

74-year-old patient presents with increased
shortness of breath. A sputum sample reveals
golden-brown beaded fibers, which result
from iron- and protein-coated fibers. On CT
scan, dense fibrocalcific plaques of the parietal
pleura are seen. A particular pneumoconiosis
is suspected. Which of the following is the
likely etiology of the patient’s condition?

A

Working in a shipyard
is associated with asbestos exposure.
Chronic inhalation of asbestos fibers can result
in asbestosis, which is marked histologically
by ferruginous bodies that stain positively with
Prussian blue. Asbestosis, unlike most other
pneumoconioses, results in marked predisposition
to bronchogenic carcinoma and to malignant
mesothelioma. Smoking and asbestos
exposure together greatly increase one’s risk of
developing bronchogenic carcinoma.

217
Q

A premature child is born in respiratory distress
and is emergently intubated. Synthetic
pulmonary surfactant is administered with no
improvement in pulmonary function. After
conducting a detailed physical examination,
the pediatrician believes that this child’s condition
is related to a failure of the embryonic
pleuroperitoneal folds to form and close. What
physical examination finding would support
the most likely diagnosis in this child?
(A) Bowel sounds in the left lower lung zone
(B) Continuous cardiac murmur
(C) Marked splenomegaly
(D) Midline deviation of the trachea

A

This child suffers
from a congenital diaphragmatic hernia
caused by the failure of the diaphragm to properly
form and close. The presence of bowel
sounds in a lung zone indicates that abdominal
contents have herniated past the boundary
of the diaphragm into the thorax. The developing
diaphragm is derived from the Septum
transversum, Pleuroperitoneal folds, Body
wall, and Dorsal mesentery of the esophagus.
These four components can be remembered
by the mnemonic “Several Parts Build the Diaphragm.”

218
Q

A 54-year-old woman complains about a persistent
cough she has had for the past three months. The cough has been bothering her a lot and making her anxious. She thinks the
anxiety is why she has lost some weight recently.
She also blames the anxiety whenever she wakes up in the middle of the night and
finds herself drenched in sweat. Further history
reveals she has rheumatoid arthritis (RA),
but her joint pains and swellings are well controlled
by medications her rheumatologist has
prescribed her. Following a physical examination,
the physician orders an x-ray of the chest. Based on the results, the physician
immediately prescribes an antibiotic regimen
and asks her to discontinue one of the drugs used to treat her RA.
The x-ray of the
chest shows a dense cavitary apical lung lesion. Which of the following
drugs increased her risk of developing
the disease

A

This patient is likely
suffering from a TB infection that was reactivated
by her use of etanercept. The x-ray of the
chest shows a dense cavitary apical lung lesion
that is highly indicative of a reactivated TB
infection. Etanercept is a fusion protein that
contains two identical tumor necrosis factor
(TNF)-receptor monomers fused to a human
IgG Fc domain. Therefore, it acts as a TNF
antagonist. In TB infections, TNF (secreted
by activated macrophages) recruits monocytes
to form the epithelioid granulomas required to
contain the mycobacteria. When TNF is effectively removed from the infection site (by drugs or other forms of immunosuppression),
patients face an increased risk of reactivation
with caseation and cavitary lesions.